Dunkle Materie und dunkle Energie sind zwei zentrale Forschungsgebiete der modernen Astronomie. Es sind auch zwei der Themen, die in der Öffentlichkeit gleichermaßen Faszination und Verwirrung hervorrufen. Das liegt einerseits an den Eigenschaften von dunkler Materie/Energie und andererseits an den falschen Vorstellungen, die darüber im Umlauf sind.

Über die dunkle Energie hab ich hier schon öfter ausführlich geschrieben. Der Nobelpreis für Physik des Jahres 2011 ging an ihre Entdecker. Auch die dunkle Materie hab ich schon öfter erklärt. Weder bei dunkler Materie noch bei dunkler Energie wissen wir derzeit, worum es sich handelt. Ihre genaue Natur ist unbekannt. Aus dieser Tatsache und der zugegebenermaßen sehr mysteriösen Bezeichnung schließen viele Menschen, dass es sich hier um reine Hirngespinste der Forscher handeln muss. Etwas, das vollkommen hypothetisch ist. Etwas, das man sich einfach nur ausgedacht hat. Vielleicht sogar nur deswegen ausgedacht hat, um irgendwie zu vertuschen, dass die ganze Astronomie hinten und vorne nicht funktioniert und nur durch die Erfindung neuer mysteriöser Dinger so halbwegs wieder zum Laufen gebracht werden kann.

Das ist natürlich Unsinn. Niemand hat dunkle Materie oder dunkle Energie einfach so „erfunden“. Beide Phänomene wurden entdeckt. Nur weil wir nicht wissen, worum es sich bei der dunklen Materie handelt, heißt das nicht, dass sie einfach so aus den Köpfen der Astronomen in die Welt geplumpst ist. Es gibt eine Vielzahl sehr handfester Beobachtungsdaten, die die Existenz der dunklen Materie nahelegen. Wissenschaftler sind sparsame Menschen; besonders wenn es um ihre Theorien geht. Je weniger fundamentale Bestandteile die Theorie hat; je weniger Annahmen gemacht werden müssen: Umso besser. Nur aus Spaß an der Freude würde man sich nicht einfach so etwas seltsames wie dunkle Materie ausdenken. Die dunkle Materie taucht deswegen in den Theorien der Astronomen auf, weil es Beobachtungen gibt, die erklärt werden müssen. Seit in den 1930er Jahren die Technik ausreichend gut geworden ist, beobachtet man überall im Universum, dass die Himmelskörper sich nicht so bewegen, wie sie es eigentlich sollten. Dafür kann es zwei Gründe geben: Entweder ist die Theorie, mit der die Bewegung berechnet wird falsch. Oder man hat etwas übersehen, dass die Bewegung beeinflusst. Beide Möglichkeiten wurden überprüft. Es wurde eine neue Theorie für die Bewegung der Himmelskörper aufgestellt, die modifizierte Newtonsche Dynamik (MOND). Mit der kann man zwar bestimmte Aspekte der falschen Bewegung korrigieren. Aber bei weitem nicht alle. Die zweite Möglichkeit ist da viel erfolgreicher. Wenn da draußen noch etwas ist, das wir übersehen haben, eben „dunkle Materie“, dann erklärt das nicht nur die „falsche“ Bewegung der Himmelskörper, sondern passt sich auch wunderbar in die bestehenden kosmologischen Theorien ein. Die dunkle Materie ist keine Erfindung, sondern die derzeit beste Interpretation von realen Beobachtungsdaten (gleiches gilt übrigens für die dunkle Energie, auch die ist keine Erfindung).

Ich komme eigentlich nur deswegen auf dieses Thema, weil ich euch den wunderbaren YouTube-Kanal von minutephysics empfehlen wollte (ich hatte letztens erst ein Video von dort verlinkt, glaube ich). Dort findet man kurze Videos, in denen aber trotzdem verständlich verschiedene physikalische Phänomene erklärt werden. Das Video über die dunkle Materie gefällt mir besonders gut:

Die dunkle Materie gehört zu den Dingen im Universum, die mich persönlich am meisten faszinieren. Ich bin mir auch ziemlich sicher, dass wir nicht mehr wahnsinnig lange warten müssen, bis wir wissen, worum es sich dabei wirklich handelt. In ein oder zwei Jahrzehnten sollten wir genügend Daten beisammen haben, um dieses Rätsel lösen zu können. Ich warte gespannt!

118 Gedanken zu „Die dunkle Materie ist keine Erfindung“
  1. Offensichtlich bist du wieder vollkommen genesen. Anders ist nicht zu erklären, dass du eine Vorlage für einen Disput lieferst, der uns Tage beschäftigen wird.

    Ich muss nur noch schnell einkaufen. Fangt bitte nicht ohne mich an 🙂

  2. Ach, und noch was: War vor kurzem nicht auch ein Artikel auf SB, in dem beschrieben wurde, dass sich die Oortsche Wolke bis auf ein Lichtjahr um die Sonne erstrecken könnte? Ich nehme an, dass wenn wir annehmen, dass jeder Stern soviel oder noch mehr Material um sich gebunden hat, wir auch damit nicht annähernd erklären können, warum sich Galaxien nicht zerlegen, richtig?

  3. @Rarehero: „Ich nehme an, dass wenn wir annehmen, dass jeder Stern soviel oder noch mehr Material um sich gebunden hat, wir auch damit nicht annähernd erklären können, warum sich Galaxien nicht zerlegen, richtig? „

    Richtig. Das wäre eine Variation der MACHO-Hypothese und die funktioniert nicht wirklich. Außerdem sind in der Oortschen Wolke zwar viele Objekte – aber die sind alle sehr klein und bei weitem nicht schwer genug, um die fehlende Materie erklären zu können.

  4. das fällt ir ja gerade erst jetzt ein…

    das nächste mal, wenn ich in einer Homöopathieoderwasauchimmer-Diskussion höre:
    „Nur weil man etwas nicht messen kann, ist es nicht automatisch nicht vorhanden“ kann ich getrost auf die dunkle Energie verweisen.

    Ist doch korrekt, oder? Ich kann sie nicht direkt messen, aber ihren Effekt (Expansion des Universus)
    Hm… das schreit fast nach einem MartinB-Artikel, was „messen“ überhaupt bedeutet.

  5. Ich weiß jetzt gerade nicht, welche Erkenntins seinerzeit den Äther überflüssig gemacht hat. Aber irgendwie habe ich das Gefühl, daß es der dunklen Energie eines Tages genauso ergehen wird.

  6. @Florian:

    Auch die dunkle Materie hab ich dunkle Materieschon öfter erklärt.

    Auch wenn es sehr viel dunkle Materie gibt, hier ist es wohl ein bisschen zu viel. 😉

  7. @Laie

    Beim Äther war es so, dass man dachte, Lichtwellen bräuchten ein Medium, so wie Schallwellen oder Wasserwellen. Wenn sich nun die Erde relativ zu diesem Medium bewegte, dann würde man ja in Richtung der Erdbewegung eine andere Lichtgeschwindigkeit messen, als quer dazu: einmal c+/-v, einmal nur c (v = Geschwindigkeit der Erde). Wie Bullet schon sagte, ergab das Michaelson-Morley-Experiment aber in jeder Richtung die selbe Geschwindigkeit.

    Im Prinzip hätte man auch schon vorher drauf kommen können: aus den Maxwellschen Gleichungen purzelte nämlich eine Konstante c in m/s raus, die keinerlei Bezugspunkt zu irgendwas hat, und mit der sich Licht ausbreiten sollte. Immer.

    Bei der dunklen Materie und dunklen Energie gibt es mehrere Gründe, sie anzunehmen: man hat Dunkle Materie auch schon von Materie getrennt gemessen (daran, wie sie mit ihrer Schwerkraft das Licht dahinterliegender Galaxien ablenkt; google mal „Bullet Cluster“). Und die Wirkung der Dunklen Energie, die das Weltall beschleunigt ausdehnt, ist eindeutig nachgewiesen und verhält sich wie eine kosmologische Konstante.

    Aber es gibt noch einen wichtigeren Grund zu ihrer Annahme: Wir können an der kosmischen Hintergrundstrahlung ablesen, dass das Weltall flach ist, d.h. es gehorcht der üblichen Euklidischen Geometrie: Dreiecke haben 180° Winkelsumme, der Kreisumfang ist 2*Pi*r (in gekrümmten Räumen wäre das nicht so). Um das Weltall flach zu machen, braucht es aber einen bestimmten Masse-/Energieinhalt pro Volumen (kritische Dichte). Und alle Materie, die wir direkt beobachten können, macht nur 5% der kritischen Dichte aus, die das Weltall flach machen könnte. Nimmt man aber die gemessenen Werte für die Dunkle Materie und Dunkle Energie hinzu (23% bzw. 72% der Dichte, der das Weltall flach machen würde), dann kommt man genau auf 100%. Und dann sollte man noch dazu wissen, dass jede kleine Abweichung von einem flachen Universum in kürzester Zeit so stark anwachsen würde, dass das Weltall eigentlich nur dann annäherend (im Rahmen der Messgenauigkeit) flach sein kann, wenn es von Anfang an extrem genau flach war.

    Das und mehr kann man in einem witzigen =>Vortrag von Lawrence Krauss hören, der mittlerweile auch als Buch erschienen ist (wartete ich schon seit Juni letzten Jahres drauf, kam Mitte Januar aber erst raus, jetzt hab‘ ich es fast durch). Krauss sagt übrigens auch, nur ein flaches Universum kann aus dem Nichts entstehen, weil die bei der Expansion geleistete Arbeit in ihm genau der potenziellen Energie entspricht, den die Materie bei der Expansion erhält.

    Für die Dunkle Materie bzw. Energie sieht es also deutlich besser aus, als für den Äther. Die Theorie verlangt sie und die Messung bestätigt sie. Nur woraus sie besteht, weiß man noch nicht.

  8. Ich möchte die vielen Videos einmal anprangern! Ich kann das abends im Gasthaus nicht hören/sehen. Ich hab viel lieber (auch Massen von) Text.

    Gebt einem armen Gastronomen die Möglichkeit, zu lernen 🙂

  9. @Alderamin, @Laie

    Zum Äther.
    Es gibt Äthertheorien, die funktionieren. Aber die sind mörder kompliziert. Einstein hat die Formeln aus so einer Theorie (von Poincare und Lorentz) genommen. und dann hat er, statt den mörder komplizierten Äther, der diese Formeln erzeugt, zu postulieren, die Formeln postuliert.
    Poincare und Lorentz hat das gar nicht gefallen, weil die etwas haben wollten, worin sich das Licht ausbreitet. Einstein war es egal, ob und worin sich das Licht ausbreitet. Er konnte die Messergebnisse vorhersagen, und das reichte ihm.
    Ob es nun einen Äther gibt, der diese Formeln erzeugt, muss der Physiker den Philosophen überlassen.
    Diese Trennung zwischen Physik und Philosophie aufgezeigt zu haben, ist Einsteins Verdienst. Einstein hat da eine Entwicklung in der Physik losgetreten, die erst in den darauf folgenden Jahrzehnten konkretisiert wurde. Bei dieser Konkretisierung war Einstein allerdings eher Bremse als Motor („Gott würfelt nicht“).

    Zur dunklen Materie.
    Es gibt da Bereiche im Universum, wo Gravitation ist, oder, anders formuliert, der Raum gekrümmt ist, ohne dass man da Materie sieht.
    Wenn man nun alles, das den Raum krümmt, als Materie bezeichnet, und der Raum nur dort gekrümmt ist, wo es Materie gibt, dann ist an diesen Stellen dunkle Materie.
    Wenn man keine dunkle Materie haben will, dann muss man irgend etwas anderes postulieren, das dann diese Raumkrümmung beschreibt.
    Beim Äther-Problem waren diese Formeln noch recht einfach. Der Äther, der das erkärt, ist überflüssiger Ballast, und so etwas lässt man nach Ockham weg.
    Bei der dunklen Materie ist es anders. Diese seltsame Raumkrümmung ist nicht mit einer relativ einfachen Formel zu beschreiben. Da ist dunkle Materie, die in einem Bereich ist, und in einen anderen Bereich nicht, das nach Ockham zu bevorzugende Modell.

  10. Man hat einen Effekt und stellt fest, dass man den Effekt mit Dunkler Energie und Dunkler Materie beschreiben kann. Gut. Aber das heißt noch lange nicht, dass beide existieren. Es könnte doch einen anderen Effekt geben, der sich derart auswirkt, wie sich Dunkle Energie und Dunkle Materie auswirken würden. Mit den Jahreszeiten ist es auch so. Der Sommer folgt immer auf den Frühling. Also könnte man schließen, dass der Frühling die Ursache des Sommers ist. Die Ursache ist aber Erdumkreisung der Sonne plus Achsenschiefstand.

    Die String Theorie soll es auch erklären können, wenn man lange genug daran herum rechnet.

    So lange man lediglich indirekte Schlüsse auf Dunkles Zeugs ziehen kann finde ich die Erklärungslage noch ziemlich dünn.

  11. @Heino

    Bis jetzt ist die Dunkle Energie im wesentlichen der Name für den beobachteten Effekt (dass der Raum sich gleichförmig und beschleunigt ausdehnt). Und sie folgt zwanglos aus der Allgemeinen Relativitätstheorie als kosmologische Konstante (allerdings mit beliebig annehmbarem Wert).

    Bei der Dunklen Materie weiß man zumindest schon einmal, dass sie klumpen kann (sich unter ihrer Gravitation verdichten), und zwar auch da, wo keine sichtbare Materie ist (wie im Bullet Cluster; von der Sorte soll es mittlerweile schon 5 bekannte Objekte geben, stand mal im Instellarum, wenn ich mich recht entsinne). Damit scheidet modifizierte Newtonsche Dynamik oder irgendetwas anderes, das an normaler Materie hängt, aus. Außerdem kann man aus der Urknalltheorie ermitteln, wieviel normale baryonische Materie entstanden sein kann, und in welchen Verhältnissen welche Elemente gebildet wurden, und das tut die Urknalltheorie für Wasserstoff bis Lithium über 12 Zehnerpotenzen, wie Krauss in seinem Buch schreibt. Und die baryonische Materie erklärt die dunkle Materie nicht, es handelt sich nicht um dasselbe Zeugs, aus dem wir bestehen, etwa in Form von erloschenen Sternen, Planeten ohne Sterne, oder dunklem Gas.

    Und, wie gesagt, ohne die Dunkle Materie fehlen uns 23% an der kritischen Dichte (und ohne die Dunkle Energie nochmal 72%), um das Weltall flach zu machen, von dem wir aber durch Messungen schon wissen, dass es flach ist.

  12. @Alderamin

    Dass die Gleichungen damit aufgehen hab ich bereits verstanden. Aber woher der Effekt kommt ist damit noch nicht erklärt. Könnte doch auch ein Effekt von außerhalb unseres Universums sein, der uns platt macht.

  13. @Heino: „Es könnte doch einen anderen Effekt geben, der sich derart auswirkt, wie sich Dunkle Energie und Dunkle Materie auswirken würden“

    Versteh ich nicht ganz. Entweder gibt es einen Effekt, dessen Interpretation dunkle Materie beinhalten. Oder es gibt einen anderen Effekt, dessen Interpretation „etwas“ beinhalte, das wie dunkle Materie aussieht? Da wir nicht wissen, was dunkle Materie ist, macht diese Aussage nicht viel Sinn. „Dunkle Materie“ ist ja per Definition das, was den beobachteten Effekt verursacht. Ob das jetzt das eine Unbekannte ist oder etwas anderes Unbekanntes, ist ziemlich egal.

    „So lange man lediglich indirekte Schlüsse auf Dunkles Zeugs ziehen kann finde ich die Erklärungslage noch ziemlich dünn. „

    Man beobachtet direkt die Auswirkungen der dunklen Materie. Man kann sie halt nur nicht sehen. Die Elektronen die aus der Steckdose kommen, kann man auch nicht sehen. Sondern nur „indirekt“ ihre Auswirkungen auf die elektrischen Geräte, die eingestöpselt werden…

  14. Es ist doch kein Wunder, dass die Leute verwirrt sind bei Formulierungen wie diese:

    Niemand hat dunkle Materie oder dunkle Energie einfach so „erfunden“. Beide Phänomene wurden entdeckt. […] Es gibt eine Vielzahl sehr handfester Beobachtungsdaten, die die Existenz der dunklen Materie nahelegen.

    Also was jetzt? Wurde es jetzt entdeckt? Oder nicht? Wie kann etwas entdeckt werden, von dem man sich nicht sicher ist, dass es existiert?

  15. @Heino

    Aber ein Effekt von außen erklärt nicht, warum dunkle Materie und normale Materie sich anziehen und Strukturen bilden. Man kann ja z.B. simulieren, wie sich die Materie in der Frühzeit des Universums zu kleinen Strukturen und Galaxien zusammengezogen hat und dabei den Mix aus sichtbarer und unsichtbarer Materie variieren. Und auch da erreicht man Strukturen, die dem sichtbaren Universum ähneln, genau dann, wenn man einen Mix aus 4/5 dunkler und 1/5 sichtbarer Materie annimmt.

    Und wenn wir mal hypothetisch annehmen, dass Dunkle Materie nichts anderes ist als normale Materie in einer höheren Dimension (etwa einer anderen Brane, die ein paar Millimeter über unserer schwebt, nur durch eine 5. Dimension von uns getrennt) und ihre Schwerkraft diese Dimension überwindet, dann ist sie immer noch eine Art Dunkler Materie. Ob sie aus supersymmetrischen Teilchen oder etwas anderem besteht, ist ja noch gar nicht ausgemacht. Aber jedenfalls verhält sie sich, wie nichtleuchtende Materie, also nennt man sie auch so. Damit hat man noch nicht viel über ihre Beschaffenheit ausgesagt.

  16. @Usul: „Wie kann etwas entdeckt werden, von dem man sich nicht sicher ist, dass es existiert? „

    Man ist sich sicher, dass es existiert. Man weiß nur nicht, um was es sich handelt. Man hat ein Phänomen entdeckt: Himmelskörper bewegen sich nicht so, wie sie es sollen. Das lässt sich nur mit der Existenz einer vorher unbekannten Form der Materie erklären. Diese Art der Entdeckung ist vielleicht nicht mit der „Entdeckung“ von Amerika o.Ä. zu vergleichen wo man ein konkretes Objekt hat auf das man zeigen und „Da ist es!“ rufen kann. Es hat ja bis heute auch noch niemand ein Elektron mit eigenen Augen gesehen. Oder ein Quark. Aber wir haben in Teilchenbeschleunigern Phänomene entdeckt, die sich nur durch die Existenz von Elektronen oder Quarks erklären lassen. So hat man Quarks entdeckt. Oder eben dunkle Materie.

  17. @Florian Freistetter

    ist das, was du sagen willst, dass der Name „Dunkle Materie“ gar nichts bedeutet? Wäre dann nicht ein Name besser gewesen, der nicht die Materie-Vorstellung hervorruft?

  18. @Heino: „Wäre dann nicht ein Name besser gewesen, der nicht die Materie-Vorstellung hervorruft? „

    Naja, die Wahrscheinlichkeit, dass es sich um „irgendetwas“ anderes handelt als Materie, ist enorm gering. Klar, man hätte es auch „BUGAR (Billiarden Unsichtbare Gigantische Alien-Raumschiffe)“ nennen können. Oder sonst irgendwie. Aber die Messungen alle auf Materie hindeuten, die dunkel ist, war „Dunkle Materie“ die bessere Wahl.

  19. @Florian Freistetter
    Wollte ich auch gerade fragen.
    Kann man denn so sicher sein, dass wir wirklich alle denkbare Materie „sehen“ können, bei den Entfernungen (und ich versteh’s nicht, das kommt erschwerend hinzu 😉

  20. @Alderamin

    #Und wenn wir mal hypothetisch annehmen,…, dann ist sie immer noch eine Art Dunkler Materie.#

    Gut. Natürlich könnte man dann Haare spalten und auch sagen, eigentlich ist es „normale“ Materie, bloss ein bisschen woanders. Aber der Begriff „Existenz“ ist eigentlich nur in unserem Universum geklärt, bzw. definiert. Ich will die Philosophie gar nicht so stark strapazieren, aber ich habe mich damit abgefunden, dass es Dinge gibt, die man vielleicht erst dann rausfindet, wenn wir alle schon tot sind. Galilei und Kepler haben es auch nicht mehr geschafft, sich ihre Fragen von Einstein beantworten zu lassen.

    Bloss Florian Freistetter verspricht uns die Lösung der Weltformel noch zu unseren Lebzeiten. Ist das nicht etwas optimistisch?

  21. @Florian Freistetter

    #Aber wir haben in Teilchenbeschleunigern Phänomene entdeckt, die sich nur durch die Existenz von Elektronen oder Quarks erklären lassen. So hat man Quarks entdeckt. Oder eben dunkle Materie. #

    Ups, hatte ich bis eben überlesen. Gibt es irgendwelche Effekte, wo man Dunkle Materie „direkt“ nachweisen kann? bzw weniger indirekt als über die Gravitation auf Galaxien?

  22. Das Problem, das die Leute mit der Dunklen Materie haben, ist offenbar, dass sie nicht zwischen den als Erklärung herangezogenen Thesen (wie z.B. den SuSy-Teilchen) und dem tatsächlich beobachteten Effekt unterscheiden.

    Dunkle Materie und Dunkle Energie sind zunächst einfach mal die Namen für Effekte, die eindeutig gemessen wurden.

    SuSy und Vakuumenergie sind Erklärungsversuche, die aber leider daran scheitern, dass sie nicht nachgewiesen (SuSy) bzw. nach bekannten Gleichung gerechnet etwas zu groß (so um die 120 Zehnerpotenzen bei der Vakuumenergie) geraten sind, weil unsere derzeitigen Kenntnisse leider nicht so weit reichen, als dass wir sie daraus herleiten könnten. Aber vielleicht liefert uns der LHC ja bald mal ein SuSy-Teilchen.

  23. @Alderamin

    #SuSy und Vakuumenergie sind Erklärungsversuche, die aber leider daran scheitern, dass sie nicht nachgewiesen (SuSy) … #

    Schon klar. Aber soweit ich das bisher verstanden hatte, hat man außer der Gravitationswirkung auch noch keine weiter gehenden Messergebnisse die man auf irgend eine unbekannte Form von Materie zurückführen konnte.

  24. Wo hat er denn etwas „versprochen“? Er glaubt (und hofft), dass die Wissenschaft in einigen Jahrzehnten eine befriedigende Erklärung haben wird – nicht mehr und nicht weniger.

  25. @Captain E.

    bitterernst wars zwar nicht gemeint, aber ein bisschen mehr wars schon:

    https://www.scienceblogs.de/astrodicticum-simplex/2008/12/neptun-vulkan-und-die-dunkle-materie.php

    #Innerhalb der nächsten 10 Jahre werden wir die dunkle Materie endgültig dingfest gemacht haben. Wir werden dann wissen, aus was sie besteht und ihre Eigenschaften genau kennen. #

    ICH habe mich damit abgefunden, dass die Lösung der echt interessanten Fragen noch etwas dauern kann. Aber falls Florian das Rätsel löst, freue ich mich natürlich und klicke deshalb öfter mal auf das scienceblog, ob ers schon rausgefunden hat
    🙂

  26. @Heino

    Ups, hatte ich bis eben überlesen. Gibt es irgendwelche Effekte, wo man Dunkle Materie „direkt“ nachweisen kann? bzw weniger indirekt als über die Gravitation auf Galaxien?

    Ja, den =>Bullet-Cluster. Da ist beim Zusammenstoß von Galaxien das Gas kollidiert und steckengeblieben, während die Dunkle Materie, die sich durch die Ablenkung des Lichts entfernterer Galaxien verrät, weitergeflogen ist. Das wäre der Fall, wenn die Teilchen der dunklen Materie selbst nur durch Schwerkraft aufeinander wirken, aber nicht durch elektromagnetische Kraft, wie das Gas.

    Wenn ich mit dem Kopf gegen die Wand knalle, dann verhindert die elektromagnetische Abstoßung der Elektronen (zusammen mit dem Pauli-Prinzip), dass ich in der Wand versinke, und so hole ich mir eine Beule. Auch Photonen wechselwirken elektromagnetisch mit den Elektronen, also werden sie absorbiert und re-emittiert, deswegen können wir mit Licht nicht durch eine Wand hindurchleuchten. So geht es dem Gas im Bullet-Cluster, es verdichtet sich, baut Druck auf, bremst so den Zusammenstoß ab.

    Ein Teilchen, dem diese Kräfte wurscht sind, z.B. ein Neutrino, würde hingegen einfach durch die Wand hindurchfliegen, als sei sie nicht da. Und bei den Teilchen der Dunklen Materie wäre es genau so, wenn sie die elektromagnetische Kraft nicht spüren, dann fliegen sie durcheinander durch und bremsen sich erst danach durch ihre Gravitation wieder ab, bis sich die Bewegung irgendwann umkehren wird. Wie bei einem Pendel.

    Das spricht dann übrigens auch dagegen, dass es normale Materie in einer anderen Brane sein könnte, denn die würde dort ja elektromagnetisch miteinander wechselwirken. War ja auch nur ein Gedankenexperiment.

  27. @Heino

    Vielleicht ist Dir der Bullet-Cluster noch nicht direkt genug, weil man hier ja auch nur die Gravitationswirkung indirekt über die Verzerrung des Raums wahrnimmt, aber mehr wird die Dunkle Materie nicht hergeben, sie ist halt mit elektromagnetischer Strahlung nicht zu fassen (Neutrinos ja auch nicht, die weist man über die schwache Wechselwirkung nach, mit deren Hilfe sie Atomkerne zerfallen lassen können, wenn sie diese genau treffen).

    Der Nachweis des durch dunkle Materie verformten Raums ist das direkteste, was astronomomische Beobachtungen hergeben werden. Nur im Teilchenbeschleuniger könnten Kandidaten für die Dunkle Materie nachgewiesen werden. Wobei diese Nachweise auch immer indirekt erfolgen über bestimmte Energiemengen, die bei den Reaktionen als ansonsten unsichtbare Teilchen davoneilen.

  28. @Heino: Hier oben sagte er aber:

    Ich bin mir auch ziemlich sicher, dass wir nicht mehr wahnsinnig lange warten müssen, bis wir wissen, worum es sich dabei wirklich handelt. In ein oder zwei Jahrzehnten sollten wir genügend Daten beisammen haben, um dieses Rätsel lösen zu können. Ich warte gespannt!

    Das hört sich wie eine ganz persönliche Meinung an, aber nicht wie ein Versprechen.

  29. @Heino

    Vielleicht ist Dir der Bullet-Cluster noch nicht direkt genug, weil man hier ja auch nur die Gravitationswirkung indirekt über die Verzerrung des Raums wahrnimmt, aber mehr wird die Dunkle Materie nicht hergeben, sie ist halt mit elektromagnetischer Strahlung nicht zu fassen (Neutrinos ja auch nicht, die weist man über die schwache Wechselwirkung nach, mit deren Hilfe sie Atomkerne zerfallen lassen können, wenn sie diese genau treffen).

    Der Nachweis des durch dunkle Materie verformten Raums ist das direkteste, was astronomomische Beobachtungen hergeben werden. Nur im Teilchenbeschleuniger könnten Kandidaten für die Dunkle Materie nachgewiesen werden. Wobei diese Nachweise auch immer indirekt erfolgen über bestimmte Energiemengen, die bei den Reaktionen als ansonsten unsichtbare Teilchen davoneilen.

  30. Ich habe vor kurzem etwas von Dunkler Strömung (Dark Flow) gehört, da ging es um Galaxienbewegung in eine Richtung, wo sie sich eigentlich nicht hinbewegen sollten (hoffe ich habe das einigermaßen richtig verstanden). Hat das auch mit Dunkler Materie und/oder Dunkler Energie zu tun, oder hat man „Dunkel“ einfach nur als Synonym für „noch nicht erklärbar“ genommen?

  31. @mr_mad_man

    Nach =>dem hier ein durchaus umstrittener Effekt möglicherweise irgendwelcher Materie, die mittlerweile nicht mehr in unserem Lichtkegel weilt, dies aber früher einmal getan hat. Also nichts in der Art der Dunklen Materie.

  32. @Alderamin
    Der Link funzt leider nicht.
    Deine Erklärungen waren übrigens klasse, und ist der Wiki Eintrag vom Bullet- Cluster neu? Kam mir beim letzten Nachsehen nicht so vielsagend vor. Na, lesen bildet ja.

  33. Dunkle Materie, beobachtet duch ein Fernrohr, und dunkle Socken, beobachtet mit einer Waage sind beides erstmal nur Messergebnisse, die man durch weitere Experimente und Instrumente genauer beschreiben muss. Freilich, bei dunklen Socken hat man auch mit Alltagsmessgeräten eine ziemlich gute Ahnung, was das ist.

  34. @Heino: „Aber soweit ich das bisher verstanden hatte, hat man außer der Gravitationswirkung auch noch keine weiter gehenden Messergebnisse die man auf irgend eine unbekannte Form von Materie zurückführen konnte. „

    Doch, es gibt Messungen von Satelliten (Fermi-Haze, Pamela,…- ist im Artikel auch verlinkt); es gibt Messungen von anderen Experimenten: https://www.scienceblogs.de/astrodicticum-simplex/2009/12/hat-man-endlich-die-dunkle-materie-entdeckt.php

    „Bloss Florian Freistetter verspricht uns die Lösung der Weltformel noch zu unseren Lebzeiten. Ist das nicht etwas optimistisch? „

    Die Weltformel ist wieder ganz was anderes. Und ich verspreche nichts. Ich denke nur, dass die Leute keine Ewigkeit brauchen werden, die dunkle Materie zu identifizieren.

  35. Eine Frage beschäftigt mich schon lange: Wenn Dunkle Materie ein real existierender Stoff ist, eben eine Form von Materie, und wenn ich hier auch lese, dass diese durchaus „klumpt“, sind dann auch Himmelskörper, Sonnensysteme, Galaxien denkbar, die quasi parallel zu den uns bekannten existieren und dennoch nicht mit diesen wechselwirken?
    Könnte, so gesehen, auch ein DM-Planet unbemerkt durch uns hindurch fliegen?
    Sorry für die laienhafte Frage, doch mein Hirn hat sie ganz ohne Beeinflussung meines Geistes formuliert und giert nach begründbaren Spekulationen…
    😉

  36. @ Florian Freistetter

    sind die [url=https://de.wikipedia.org/wiki/WIMP]WIMPs[/url] denn der momentan bevorzugte Erklärungsversuch für die dunkle Materie? Im Wikipedia Artikel steht auch noch was über Axionen, Machos und dunklen Kräften, die sich, soweit ich es verstanden habe, als Alternativen darstellen, also eins das andere ausschließt.

  37. @Jochen O aus S

    Nein, mit „klumpen“ meinte ich, dass sie sich verdichten kann, aber sie haftet nicht aneinander, sie hat keine Van-der-Vaals-Kräfte und keine Elektronenbindungen, weil das alles elektromagnetische Kräfte sind. Sie hat nur Gravitation (die ist aber viel zu schwach, um Partikel aneinander zu binden) und schwache Wechselwirkung (hat nur kurze Reichweite und ist ebenfalls sehr schwach). Außerdem erwartet man von einigen Kanditatenteilchen (siehe Kommentar von Bjoern), dass sie ihre eigenen Antiteilchen sind und „annihilieren“ (sich in Energie umwandeln), wenn sie miteinander zusammenstoßen.

    Also bleibt dunkle Materie ein „ätherischer Stoff“, aus dem keine feste Materie entstehen kann. Der allerdings wesentlich ist, um normale Materie zu Galaxien und größere Strukturen formen zu lassen.

  38. Wobei ich selbst noch nicht verstanden habe, wie die Dunkle Materie es schafft, sich zu verdichten.

    Wenn normales Gas unter seiner Schwerkraft kollabiert, dann steigt der Druck, die Teilchen stoßen zusammen, werden dabei schneller und somit heißer. Dadurch beginnen sie, zu strahlen, und so werden sie den Großteil ihrer Bewegungsenergie los, so dass der Kollaps fortlaufen kann.

    Die dunkle Materie kann aber nicht strahlen und dürfte (bis auf Anihilationsprozesse) auch nicht zusammenstoßen können, also müsssten die Teilchen aufeinander zu stürzen, durcheinander durch fliegen und wieder auseinanderstreben, bis sie sich durch ihre Gravitation so weit abgebremst haben, dass sie wieder aufeinander zurückfallen und so weiter, ad infinitum. Weiß jemand, wie die Dunkle Materie ihre Bewegungs-/Potenzielle Energie loswerden kann?

    @Heino: Axionen sind WIMPs (Weakly Interacting Massive Particles, also Schwere Teilchen, die die Schwache Kernkraft spüren). MACHOs wären kleine Asteroiden, ausgebrannte Sterne und dergleichen (Massiv Compact Halo Objects), aber die können es wegen der begrenzten Zahl der Baryonen, die beim Urknall entstanden sein können, nicht sein, jedenfalls nicht in der Mehrzahl.

  39. @Myself

    Wobei ich selbst noch nicht verstanden habe, wie die Dunkle Materie es schafft, sich zu verdichten.

    Hab‘ mal ein wenig gegoogelt, anscheinend reichen gravitative Merhkörper-Interaktionen zur Abkühlung von DM-Wolken aus. Einige heiße Teilchen werden herauskatapultiert, der Rest kühlt ab. Im Prinzip so was ähnliches wie Verdunstungskälte, nur eben rein gravitativ.

    Siehe z.B. =>hier und =>hier.

  40. Annihilation der WIMPs , Bullet-Cluster/Abell 520 und Pamela …
    Selbst unter Berücksichtigung von nur ‚wenig‘ Wechselwirkung der WIMPs selbst untereinander – warum bleibt „Dunkle Materie“(WIMPs) eigentlich gravitativ gebunden und fliegt nicht einfach auseinander ohne andererseits ‚zusammen zu stürzen‘? – sollten nicht Kollisionen bspw. bei Abell 520 trotz allem eine erhöhte Rate aufweisen und so u.U. von Pamela ( Annihilation, gamma-Emmission) registriert werden können?

  41. @Alderamin:

    „Weiß jemand, wie die Dunkle Materie ihre Bewegungs-/Potenzielle Energie loswerden kann?“

    Genau dann, wenn _diese_ Frage beantwortet sein wird, erhält die Vorstellung von umhersausender „Quasi“-Masse als echte Erklärung des DM-Effektes einen Sinn…

    „Bei der Dunklen Materie weiß man zumindest schon einmal, dass sie klumpen kann (sich unter ihrer Gravitation verdichten), und zwar auch da, wo keine sichtbare Materie ist (wie im Bullet Cluster; von der Sorte soll es mittlerweile schon 5 bekannte Objekte geben, stand mal im Instellarum, wenn ich mich recht entsinne). Damit scheidet modifizierte Newtonsche Dynamik oder irgendetwas anderes, das an normaler Materie hängt, aus.“

    Inwiefern widerspricht das einer modifizierten Gravitation? Könntest Du das bitte näher erläutern?

    Vielen Dank im Voraus.

  42. @SCHWAR_A

    Inwiefern widerspricht das einer modifizierten Gravitation? Könntest Du das bitte näher erläutern?

    Es gibt zwei Ideen zur Erklärung der Rotationskurven von Galaxien:

    1) Die Gravitation der sichtbaren Materie nimmt über große Entfernungen nicht mit dem Quadrat der Entfernung ab, sondern langsamer (MOND).
    2) Es gibt unsichtbare Materie, welche den Großteil der Gravitation verursacht.

    Im Fall 1 tritt Gravitation da auf, wo die sichtbare Materie ist. Im Fall 2 tritt sie da auf, wo sichtbare und/oder dunkle Materie ist. Da es mehr dunkle als sichtbare Materie geben soll, würde die Gravitationswirkung eher an den Orten festzustellen sein, wo dunkle Materie sich konzentriert (nur befindet sich üblicherweise dort auch die sichtbare Materie, die der Gravitation der dunklen Materie folgt).

    Im Bullet-Cluster haben sich sichtbare und dunkle Materie bei einem Zusammenstoß zweier Galaxienhaufen getrennt. Man sieht große Gravitationswirkung da, wo wenig sichtbare Materie ist, und wenig Gravitationswirkung, wo der Großteil des sichtbaren Materie (in diesem Fall das Plasma zwischen den Galaxien) ist. Folglich ist die 2. Erklärung die richtige, denn die 1. würde verlangen, dass die Gravitation immer an die sichtbare Materie gebunden sein muss.

    q.e.d.

  43. @Alderamin:

    Danke.
    „Man sieht große Gravitationswirkung da, wo wenig sichtbare Materie ist, und wenig Gravitationswirkung, wo der Großteil des sichtbaren Materie (in diesem Fall das Plasma zwischen den Galaxien) ist.“

    Bei modifizierter Gravitation heben sich doch ebenso die Wirkungen zwischen den Galaxien gegenseitig auf, also genauso wie es beobachtet wird.

    Eigentlich sieht man auf den BM-DM-Bildern DM-Halos um beide Galaxien, nur daß die Wirkung zwischen beiden Galaxien in unterschiedliche Richtungen zeigen, also Photonen der Backgroundsterne _nicht_ verbogen werden.

  44. @StefanL

    Selbst unter Berücksichtigung von nur ‚wenig‘ Wechselwirkung der WIMPs selbst untereinander – warum bleibt „Dunkle Materie“(WIMPs) eigentlich gravitativ gebunden und fliegt nicht einfach auseinander ohne andererseits ‚zusammen zu stürzen‘?

    Weil sie beim Urknall in Ruhe befindlich entstand („Cold Dark Matter“ – kalt heißt, die Teilchen bewegen sich nur langsam). Die Verteilung war anfangs sehr gleichförmig, zufällige Dichteschwankungen waren jedoch vorhanden. Wo höhere Dichte war, wirkte diese als Attraktor und zog mehr Dunkle Materie an. Die Dunkle Materie wurde beschleunigt und fiel auf diese Dichtezonen zu und druchdrang diese, um wieder auseinanderzustreben, aber da sie aus einem Ruhezustand startete, kann sie höchstens wieder einen Ruhezustand erreichen, um dann wieder zurückzufallen. Deswegen kann sie nicht auseinanderfliegen.

    Kollabieren kann sie auch nicht, denn am Ort der größten Dichte sind die Teilchen zu schnell und entkommen wieder. Sie haften nicht aneinander, sie können keine Bewegungsenergie abstrahlen, also können sie nicht langsamer werden, außer durch erneute Expansion.

    Am Ende verdichten sie sich doch lokal durch den oben beschriebenen Abkühlmechanismus: bei zufälligen Interaktionen werden manche Teilchen beschleunigt, andere langsamer. Die beschleunigten sind dann aber so schnell, dass sie die Fluchtgeschwindigkeit des DM-Haufens überschreiten und für immer entkommen (wenn sie aus dem Ruhezustand starteten, dann hatten sie bei der größten Annäherung durch den freien Fall dorthin schon fast Fluchtgeschwindigkeit, d.h. ein kleiner Geschwindigkeitszuwachs reicht aus, um sie entkommen zu lassen). Die anderen Teilchen bleiben zurück und haben weniger Energie, also entfernen sie sich weniger vom Zentrum des Haufens als vorher. Und so kann der DM-Haufen allmählich abkühlen und schrumpfen, durch eine Art Verdunstungsprozess. So habe ich das verstanden, scheint mir ein plausibler Mechanismus zu sein.

    sollten nicht Kollisionen bspw. bei Abell 520 trotz allem eine erhöhte Rate aufweisen und so u.U. von Pamela ( Annihilation, gamma-Emmission) registriert werden können?

    Könnte mir folgende Erklärung vorstellen: zum einen sind die DM-Zonen da schon durcheinander durch geflogen und die Interaktionen weitgehend Vergangenheit. Zum anderen erzeugt das noch kollidierende Gas mächtig Röntgenstrahlung, vielleicht überlagert das die Annihilationsstrahlung. Der Cluster ist außerdem ziemlich weit weg. Man wird wohl eher Annihilationsstrahlung aus dem Zentrum der Milchstraße messen.

  45. @SCHWAR_A

    Eigentlich sieht man auf den BM-DM-Bildern DM-Halos um beide Galaxien, nur daß die Wirkung zwischen beiden Galaxien in unterschiedliche Richtungen zeigen, also Photonen der Backgroundsterne _nicht_ verbogen werden.

    Eine exakte Kompensation der Gravitation der zwei Galaxienhaufen wäre aber nur auf einer schmalen Fläche gegeben, ich denke mal, die Leute, die die Messungen interpretiert haben, werden das schon berücksichtigt haben. Kannst Herrn Douglas Clowe ja mal direkt anschreiben und fragen, =>Autor und Quelle sind im Bullet-Cluster-Wikipedia-Eintrag verlinkt.

  46. @Alderamin:

    Stell Dir mal vor, die DM-Effekt-Überlagerung der Gravitation sei linear.

    Dann erhälst Du genau dieses Bild: weiträumig ist der Zwischenraum ohne DM-Effekt-Wirkung.

  47. @SCHWAR_A

    Abnehmen muss die Schwerkraft mit der Entfernung ja schon, auch wenn sie linear abnimmt, und dann kann sie sich nur da ausgleichen, wo auf dem Weg von dem einen Haufen zum anderen die Schwerkraft des einen so weit abgenommen hat, wie die des anderen zugenommen hat. Davor und dahinter linst mal der eine, mal der andere Galaxienhaufen. Und außerhalb der Verbindungsachse linsen sogar beide zusammen in die gleiche Richtung.

    Aber wie gesagt, frag‘ nicht mich, das mit der DM war ja nicht meine Idee, frag‘ Douglas Clowe oder einen Ko-Autor. Den Link hast Du.

  48. @Alderamin:

    Abnehmen der Gravitation:

    Klar, aber hier geht es ja um das „Add-On“ zur Gravitation, den DM-Effekt.
    Und der könnte durchaus weiträumig linear sein.

  49. Mein Eindruck bleibt, dass die Probleme, die zum Nachweis der DM zu lösen sind, kein Pappenstiel sind. Da wird noch einiges an Rechenleistung verbraucht werden. Ich bleibe skeptisch und warte mal ab. Vielleicht erhärtet sich das mit den WIMPs, oder vielleicht findet jemand eine andere/neue Theorie, die mehr erklärt.

    Florian wird uns auf dem Laufenden halten 🙂

    Ach noch ne Frage: auf der DM-Karte, kann man da erkennen, ob in unserer Gegend auch eine DM Konzentration ist? Oder herrscht hier DM-Leere??

  50. @SCHWAR_A

    Ich nehme an, Du meinst nicht linear (Abnahme mit 1/r) sondern konstant. Nein, das geht nicht, dann könnte nichts linsen, dazu braucht es einen Schwerkraftgradienten. Außerdem ist das Graviationsfeld ein Vektorfeld, das kann schon per se nicht konstant sein, weil sich die Richtung mit dem Ort ändert.

  51. @Heino

    Ach noch ne Frage: auf der DM-Karte, kann man da erkennen, ob in unserer Gegend auch eine DM Konzentration ist? Oder herrscht hier DM-Leere??

    Da die Milchstraße wie jede andere zu schnell rotiert für ihre sichtbare Masse, wird auch hier Dunkle Materie sein, die haloförmig die Milchstraße umgibt, im Zentrum dichter, nach außen hin dünner werdend. Wenn die WIMP-Theorie richtig ist, fliegt sie gerade durch Dich durch, so wie die Sonnenneutrinos, denn da sie die elektromagnetische Kraft nicht spürt, bist Du für sie im wesentlichen ein feldfreies Vakuum mit einigen sehr dünn verteilten Atomkernen und Elektronen, mit denen sie nur interagieren kann, wenn eines ihrer Teilchen zufällig genau eines davon mit der richtigen Energie trifft. Und das kommt so gut wie nie vor, sonst hätten wir die Teilchen der Dunklen Materie längst nachgewiesen.

  52. Zitat Florian:
    “ Außerdem sind in der Oortschen Wolke zwar viele Objekte – aber die sind alle sehr klein und bei weitem nicht schwer genug, um die fehlende Materie erklären zu können.“

    Aber wenn ich das als Laie richtig verstehe sind sie trotzdem ein TEIL der fehlenden Materi, richtig?

  53. @Alderamin – Danke für die Erläuterung,
    einsichtig ist es für mich allerdings immer noch nicht. ‚cold‘ mit ‚langsam‘ zu assoziieren hab‘ ich kein Problem. Mit dem Urknall zu argumentieren so, dass da ‚Teilchen‘ enstanden sind (so gut wie) ohne Bewegungs(/Dreh)impuls? Ich weiß nicht…und schließlich bewegen sich diese ‚dark matter‘-Klumpen ja doch auch mit den Geschwindigkeitsvektoren der Galaxien zu denen sie gehören (die ‚pink‘ farbenen Teile in den Bildern sind ja Gase/Plasma und nicht der ganze sonstige baryonische Rest der Galaxien). wenn es anders ist/wäre, kann man dann nicht Effekte auf die Rotationsgeschwindigkeit einer Galaxie( die quasi ihre ‚dunkle Materie‘ verloren hat) beobachten?
    Bewegung hin zu Dichteschwankungen – hindurchgehen und gewissermaßen zurück-pendeln – gut. Dabei in relativ kurzem Zeitraum die (großräumige) Struktur(Galaxienverteilung) des Universums festlegen andererseits aber – immerhin ist die gravitative Wechselwirkung zu baryonischer Materie ja vorhanden(‚Galaxien-stabilisierung‘) – dann doch keine Tendenz in den zentralen Schwarzen Löchern der Galaxien zu ‚enden‘ bei mehreren Milliarden Jahren Zeit (insbesondere, mit kaum Eigengeschwindigkeit)?
    Hmm – der „Verdunstungsprozeß“ führt dazu, dass die DM-Haufen verschwinden? Bewegungsenergie(-(abgabe): Gilt hier ebenfalls E=mc² ? ..und die Frage nach ‚Gravitonen‘ und somit auch Energieübertragung?
    So wie ich das verstanden habe ist Abell 520 doch noch (zumindest teilweise) mitten in der Kollision – da könnte dann vielleicht tatsächlich nur das Argument ‚unterhalb der Nachweisgrenze‘ ziehen… bei der Menge beteiligter ‚Teilchen‘ ist doch dann eine gewisse Skepsis gegenüber ‚kleinen‘ Laborversuchen noch durchaus angebracht, oder widerlegt mich da die Statistik bereits( dieses 1:4 ; 25% – Fehlerwahrscheinlichkeit kanns ja als Bestätigung nicht sein)?
    Wenn der Begriff „exotische Materie“ nicht schon vergeben wäre…

  54. @StefanL

    Mit dem Urknall zu argumentieren so, dass da ‚Teilchen‘ enstanden sind (so gut wie) ohne Bewegungs(/Dreh)impuls? Ich weiß nicht…und schließlich bewegen sich diese ‚dark matter‘-Klumpen ja doch auch mit den Geschwindigkeitsvektoren der Galaxien zu denen sie gehören

    Die Galaxien sind sozusagen die Schaumkronen auf den Wellen der dunklen Materie (das ist eine Metapher, die Wellen bitte nicht wörtlich nehmen). Die Dunkle Materie bewegt sich so schnell, wie sie durch freien Fall aus großer Entfernung geworden ist, ihre Klumpen vollführen komplexe Orbits umeinander, die Galaxien mit ihr. Die Teilchen waren aber ursprünglich gleich verteilt und sind aus der Energie des Urknalls auskondensiert, die den ganzen Raum erfüllt hat, wie auch die normale Materie. Da waren sie noch in relativer Ruhe. Das sagt das Cold Dark Matter Modell.

    wenn es anders ist/wäre, kann man dann nicht Effekte auf die Rotationsgeschwindigkeit einer Galaxie( die quasi ihre ‚dunkle Materie‘ verloren hat) beobachten?

    Die Galaxien im Bullet-Cluster haben ihre Dunkle Materie nicht verloren, die haben genau das getan, was die Dunkle Materie auch getan hat, sie sind aneinander vorbeigeflogen, weil zwischen ihnen viel Platz ist. Nur das intergalaktische Gas der Galaxienhaufen ist kollidiert, weil es den ganzen Raum erfüllt.

    dann doch keine Tendenz in den zentralen Schwarzen Löchern der Galaxien zu ‚enden‘ bei mehreren Milliarden Jahren Zeit (insbesondere, mit kaum Eigengeschwindigkeit)?

    Nein, Schwarze Löcher werden immer überschätzt, das sind keine Staubsauger. Die sind sehr klein, und alles, was sie nicht zufällig geradlinig trifft, wird einfach beschleunigt und fällt auf einer Hyperbel -oder Parabelbahn an ihnen vorbei. Das schwarze Loch im Zentrum der Milchstraße ist im Verhältnis zur Milchstraße vielleicht so groß wie eine Cent-Münze im Vergleich zu den USA (habe ich mal in Sky&Telescope gelesen). Damit dieses Schwarze Loch Dunkle Materie schluckt, muss diese das Schwarze Loch genau treffen. Jetzt überlege mal, wie lange so was dauern würde, wenn sich die Dunkle Materie auf einem Volumen mit dem Durchmesser der USA verteilen würde, wenn das Ziel so klein ist wie eine Münze.

    Hmm – der „Verdunstungsprozeß“ führt dazu, dass die DM-Haufen verschwinden? Bewegungsenergie(-(abgabe): Gilt hier ebenfalls E=mc² ? ..und die Frage nach ‚Gravitonen‘ und somit auch Energieübertragung?

    Der Verdunstungsprozess führt dazu, dass die DM-Haufen dichter und kühler werden. Ganz verdunsten werden sie dabei nicht, die mittlere Geschwindigkeit wird immer kleiner, und mit zunehmender Dichte nimmt die Fluchtgeschwindigkeit an den Außenrändern immer mehr zu, es wird für die Teilchen immer schwerer, einem DM-Haufen zu entkommen.

    E=mc^2 hat nix damit zu tun (nur bei Annihilation bzw. bei der Entstehung der Materie/dunklen Materie beim Urknall), wie kommst Du jetzt darauf?

    Gravitonen: sollen die (noch nicht nachgewiesenen) Austauschteilchen der Gravitation in einer Großen Vereinheitlichenden Theorie sein und mithin auf Materie wie dunkle Materie wirken, die muss man hier aber nicht bemühen, es reicht das Einsteinsche Modell der Raumkrümmung. Eigentlich reicht sogar Newton, weil die dunkle Materie sich langsam bewegt und dünn verteilt ist.

    bei der Menge beteiligter ‚Teilchen‘ ist doch dann eine gewisse Skepsis gegenüber ‚kleinen‘ Laborversuchen noch durchaus angebracht, oder widerlegt mich da die Statistik bereits( dieses 1:4 ; 25% – Fehlerwahrscheinlichkeit kanns ja als Bestätigung nicht sein)?

    Verstehe jetzt nicht, worauf sich das bezieht. Welche Versuche, welche Fehlerwahrscheinlichkeit?

  55. @Alderamin
    Das mit den 25% hab‘ ich aus Florians (08.02.12 · 09:28 Uhr) link:
    https://www.scienceblogs.de/astrodicticum-simplex/2009/12/hat-man-endlich-die-dunkle-materie-entdeckt.php
    E=mc² – ist ja für ’normale‘ Materie gültig ( eigentlich müßte man ja von ’nicht-normal‘ reden bei den Mengen im Universum 😉 ) – und so ist es tatsächlich nur die Frage ob dies für die ‚Massen‘ der ‚Dunklen Materie‘ eben auch gilt …und so eben ‚Teilchengrößen‘ bestimmt/abgeschätzt werden können (ggfs. eben auch Bewegungsenergieanteile/ Gleichgewichtszustände usf. und sie so ganz nebenbei gleich wieder etwas normaler machen würde 😉 ).
    Ja, in Betracht der Mengenverhältnisse hat die CDM wohl die ’normale‘ Materie ‚mit sich gezogen‘ und so an den ‚Dichtepunkten‘ für die Galaxienbildung(mit i.d.R. einem SL in der Mitte) gesorgt. Also so ganz sehe ich da noch nicht warum die Beschleunigung/Fallen in Richtung Mittelpunkt des ‚Schwerkraftgenerators‘ dann nicht ins SL weisen. Soll da tatsächlich die ‚Ablenkung‘ durch andere (schwächere) Gravitationsquellen die Bewegung/Fallen der CDM so stark beeinflußt haben um Hyperbeln/Parabeln zu generieren? Versteh‘ mich nicht falsch – USA und Cent sind schon beeindruckend aber Dichteschwankungen (im Urplasma?) mit derartig phänomenalen Resultaten deuten doch auch auf entsprechende ‚Kräfte‘ hin. Lassen sich eigentlich Schwankungen in den DM-Halos von Galaxien ausmachen oder ist ein DM-Halo in sich homogen – also u.U. mehrere ‚Klumpen in einer Galaxis oder nur einer? Folgt(/bewirkt) der DM-Halo der Eigenrotation einer Galaxis oder ist er nur eine umgebende ‚diffuse‘ Wolke?
    Gravitonen: es geht mir dabei nicht um die Beschreibung(Newton, ART) sondern falls sich ‚Gravitation‘ per ‚Graviton‘ vermittelt so ist dies ja tatsächlich ein Austauschteilchen(/Energieüberträger) der WIMPs…rein geometrisch braucht’s das nicht…

  56. @Alderamin:

    „Ich nehme an, Du meinst nicht linear (Abnahme mit 1/r) sondern konstant.“

    Nein, ich meine eine weitreichend lineare Zunahme der Gravitationskonstante nach dem Muster

    G(r) = G_0 · (1 + X_1·r·e^(-Y_1·r))

    Das ist eine mögliche „modified gravitation“ – auch für Vektorfelder, wenngleich diese hiermit nicht mehr so einfach zu berechnen sind.
    Simulationen mit BM-only sollten damit aber umgehen können…

  57. a propo USA und cent-Münze – nur Spekulation: … da komm‘ ich so grob auf 10e25 (Jahre) und das wäre dann noch unterhalb der ‚magischen‘ Grenze von 10e32 so, daß ggfs. tatsächlich die DM im (Zentrums-)SL verschwunden wäre bevor die Protonen zerfallen wären – aber das ist natürlich pure (fehlerbehaftete) Spekulation und keinesfalls als abgesicherte Rechnung zu sehen…

  58. @StefanL

    E=mc² – ist ja für ’normale‘ Materie gültig ( eigentlich müßte man ja von ’nicht-normal‘ reden bei den Mengen im Universum 😉 ) – und so ist es tatsächlich nur die Frage ob dies für die ‚Massen‘ der ‚Dunklen Materie‘ eben auch gilt

    E=mc^2 gilt für jede Art von Materie, auch für die exotischsten Partikel, die in Teilchenbeschleunigern erzeugt wurden. Bzw. für die Ruhmasse von Teilchen, die eine solche haben. Allgemein gilt E^2=m^2c^4 + p^2c^2 wobei p der Impuls ist. Das gilt auch für Photonen.

    Also so ganz sehe ich da noch nicht warum die Beschleunigung/Fallen in Richtung Mittelpunkt des ‚Schwerkraftgenerators‘ dann nicht ins SL weisen. Soll da tatsächlich die ‚Ablenkung‘ durch andere (schwächere) Gravitationsquellen die Bewegung/Fallen der CDM so stark beeinflußt haben um Hyperbeln/Parabeln zu generieren?

    Wegen Kepler 1. Die Bahnen im Schwerefeld um ein Massezentrum haben dieses IMMER in einem Brennpunkt der Bahn (der Ellipse [Exzentrizität < 1], Parabel [Exzentrizität = 1] oder Hyperbel [Exzentrizität > 1], was anderes geht nicht). Nur eine sehr, sehr enge Parabel oder Ellipse bzw. eine sehr gering abknickende Hyperbel kommen nahe genug an ein ausgedehntes Objekt heran, um mit ihm kollidieren zu können. Damit eine Parabel eng werden kann, muss man von Anfang an schon fast genau auf die Zentralmasse zielen, damit man diese höchstens knapp verfehlt. Wenn die Bahn zunächst vorbei geht, hat man schon verloren, dann kann nur noch eine weite Parabel daraus werden. Ich hoffe, das ist einigermaßen anschaulich erklärt, ansonsten lies Dich mal ein wenig in die Keplerschen Gesetze und das Newtonsche Gravitationsgesetz ein. Stichwort Erhaltung von Drehimpuls und der Summe aus Potenzieller und Bewegungsenergie.

    Lassen sich eigentlich Schwankungen in den DM-Halos von Galaxien ausmachen oder ist ein DM-Halo in sich homogen – also u.U. mehrere ‚Klumpen in einer Galaxis oder nur einer? Folgt(/bewirkt) der DM-Halo der Eigenrotation einer Galaxis oder ist er nur eine umgebende ‚diffuse‘ Wolke?

    Man kann wohl feststellen, ob sich die Dunkle Materie zum Zentrum einer Galaxie hin konzentriert (das tut sie) und dass sie sich kugelförmig um die Galaxie legt, aber keine Klumpen feststellen. Man kann auch nicht ihre Bewegung messen. Es ist aber davon auszugehen, dass sie ebenfalls einen Nettodrehimpuls in Richtung der Galaxie hat, denn die Materie, die die Galaxie gebildet hat, hat sich ja zusammen mit der Dunklen Materie bewegt. Allerdings flacht sich Materie dann immer zu einer Scheibe mit Kreisbahnen ab, da Material, das geneigt zur Scheibe oder auf elliptischen Bahnen in dieser rotiert, mit Scheibenmaterial kollidiert und somit gebremst wird. Für die Dunkle Materie gilt das aber nicht, die kann alle möglichen Bahnen um den Massenmittelpunkt beschreiben, in allen Neigungen und mit allen möglichen Exzentrizitäten. Daher flacht sie sich nicht zur Scheibe ab.

  59. Alderamin schrieb (07.02.12 · 20:34 Uhr):

    > Wenn sich nun die Erde relativ zu diesem Medium bewegte, dann würde man ja in Richtung der Erdbewegung eine andere Lichtgeschwindigkeit messen, als quer dazu: einmal c+/-v, einmal nur c (v = Geschwindigkeit der Erde).

    „c + v“??
    Falls sich die Erde in einer Region befand, deren Brechungsindex den Wert „n“ hatte
    (… Wie würde man das eigentlich messen? …)
    und sich die Erde dabei mit Geschwindigkeit „v“ gegenüber einem (dem?) Inertialsystem bewegte, dessen Mitglieder Lichtgeschwindigkeit als „in alle Richtungen gleich, c/n“ feststellten,
    dann sollte und würde man die entsprechenden Lichtgeschwindigkeits-Werte (d.h. gegenüber Mitgliedern des Inertialsystems zu dem die Erde zumindest näherungsweise gehörte) ja wohl stattdessen als „längs“
    „(c/n +/- v) / (1 +/- 1/n v/c)“
    bzw. „quer“
    „Sqrt[ (c/n)^2 – v^2 ] / Sqrt[ 1 – (v/c)^2 ]“
    bewerten.

    > das Mich[elson]-Morley-Experiment [ergab] aber in jeder Richtung die selbe Geschwindigkeit.

    Daraus wäre offenbar zu schließen, dass der Brechungsindex in der entsprechenden Versuchsregion den Wert n = 1 hatte.
    (Allerdings:
    welche geometrischen Beziehungen zwischen Elementen der Versuchsanordnung, z.B. bestimmte Bedingungen an „Armlängen“, wurden dabei eigentlich von vornherein gefordert, bzw. wie wurde entschieden, ob diese Bedingungen erfüllt waren?)

    > [In] der üblichen Euklidischen Geometrie: Dreiecke haben 180° Winkelsumme, der Kreisumfang ist 2*Pi*r

    Das stimmt zwar; fraglicher ist allerdings, was z.B. unter „Winkel“ zu verstehen und zu ermitteln wäre, falls man (noch) nicht wüsste, ob in einem bestimmten Versuch „Euklidische Geometrie zutraf“, oder nicht, sondern man das erst herausfinden wollte. Falls man z.B. zunächst einmal versuchen müsste herauszufinden und zu quantifizieren, ob bestimmte Markierungen „auf einem Winkelmesser“ zueinander eben wären (oder inwiefern nicht), oder welche Beteiligten bzw. welche Markierungen „auf einer Geraden zwischen“ zwei gegebenen „Ecken“ (eines „Dreiecks„) „gelegen“ hätten, und welche nicht.

    Eine geeignete Verallgemeinerung der üblichen Euklidischen Geometrie besteht z.B. in „metrischen Räumen“, d.h. indem man alle geometrischen Beziehungen auf (gegebene) Distanzverhältnisse zwischen den identifizierbaren Beteiligten zurückführt. Dabei gelten fünf gegebene Beteiligte als zueinander „flach“ (bzw. vier Beteiligte als zueinander „eben“, bzw. drei Beteiligte als zueinander „gerade“) falls die (jeweilige) „Cayley-Menger-Determinante“ ihrer Distanzverhältnisse gleich Null ist.

  60. @Alderamin:

    „Man kann wohl feststellen, ob sich die Dunkle Materie zum Zentrum einer Galaxie hin konzentriert (das tut sie)“

    Wo hast Du das denn her?
    Meines Wissens ergibt sich eine Zunahme an DM erst zur Halo hin und fällt weiter außen wieder ab – also eine Konzentration nur im Halo-Gebiet.

  61. @Thomas

    Zu Kroupa und seiner Theorie, siehe =>hier

    @SCHWAR_A

    Z.B. =>hier: „Galaxies show signs of being composed largely of a roughly spherically symmetric, centrally concentrated halo of dark matter with the visible matter concentrated in a disc at the center.“

    Meines Wissens ergibt sich eine Zunahme an DM erst zur Halo hin und fällt weiter außen wieder ab – also eine Konzentration nur im Halo-Gebiet.

    Der Halo ist ein die Milchstraße kugelförmig einhüllender Bereich, der auch die Milchstraße selbst und ihr Zentrum beinhalt. Er ist keine Schale.
    @Frank Wappler

    dann sollte und würde man die entsprechenden Lichtgeschwindigkeits-Werte (d.h. gegenüber Mitgliedern des Inertialsystems zu dem die Erde zumindest näherungsweise gehörte) ja wohl stattdessen als „längs“
    „(c/n +/- v) / (1 +/- 1/n v/c)“
    bzw. „quer“
    „Sqrt[ (c/n)^2 – v^2 ] / Sqrt[ 1 – (v/c)^2 ]“
    bewerten.

    Mag sein, aber die Details ändern nichts an der Grundaussage, dass die gemessene Geschwindigkeit im Äther in Längs- und Querrichtung nicht dieselbe wäre, weil die Erdbewegung mit ins Spiel käme.

    Das stimmt zwar; fraglicher ist allerdings, was z.B. unter „Winkel“ zu verstehen und zu ermitteln wäre, falls man (noch) nicht wüsste, ob in einem bestimmten Versuch „Euklidische Geometrie zutraf“, oder nicht, sondern man das erst herausfinden wollte.

    Worum es geht, ist, dass in der kosmischen Hintergrundstrahlung, die 380000 Jahre nach dem Urknall entstand, keine Korrelationen über mehr als 380000 Lichtjahre existieren dürften. Misst man nun die Feinstruktur der Hintergrundstrahlung nach und ermittelt den größten Winkelabstand, in dem noch Korrelationen zu messen sind, kommt man auf etwa 1° (und der Sichtwinkel ist wohldefiniert). Das ist genau der Wert, den man bei euklidischer Geometrie erwarten würde. In positiv gekrümmten Räumen wäre der Winkel größer, in negativen kleiner. Das betrachtete Dreieck ist gleichschenklig und besteht hier aus der Grundlinie in der kosmischen Hintergrundstrahlung und dem Sichtwinkel beim Beobachter.

  62. @Alderamin:

    „Er [Halo] ist keine Schale“

    OK, das war mir nicht bewußt, da alle Bilder nur die Umgebung (Schale) intensiv blau zeigen, aber Richtung Zentrum immer blasser werden.

    Wenn aber tatsächlich die DM-Konzentration Richtung Zentrum immer größer werden sollte, dann hätte ich doch dort auch die stärkste Wirkung, oder nicht? Also eine nicht mehr zum G_0/r² passende Gravitation nahe der BM.

    Sowas paßt nur dann zusammen, wenn man in Simulationen zuerst KEINE BM verwendet, sondern NUR DM, diese dann „laufen“ läßt und anschließend in den dichtesten Punkten DM durch BM austauscht.

    Die Natur macht das aber nicht so! Da wirken BM und DM zur gleichen Zeit miteinander. Das bedeute aber, daß man in den simulationen von den inneren Halo-Regionen die Wirkung der plazierten BM subtrahieren muß, um die tatsächliche Verteilung zu erhalten. Dadurch erhält man dann etwas Schalenähnliches.

    Gravitationslinsen-Messungen würden sonst auch eine viel stärkere Verzerrung zeigen, je näher man der BM kommt…

  63. @SCHWAR_A

    Wenn aber tatsächlich die DM-Konzentration Richtung Zentrum immer größer werden sollte, dann hätte ich doch dort auch die stärkste Wirkung, oder nicht? Also eine nicht mehr zum G_0/r² passende Gravitation nahe der BM.

    Du hast immer die Gravitationswirkung der in einer gedachten Kugelschale, die Deinen Ort berührt, enthaltenen Masse. Wenn die Masse kugelsymmetrisch verteilt ist, spielt es dabei keine Rolle, ob sie in einem Punkt im Zentrum konzentriert ist, den Raum gleichförmig erfüllt, nach innen dichter wird, oder wie auch immer. Die Milchstraße ist zwar nicht kugelförmig, aber innerhalb der Scheibe spielt das aus Symmetriegründen keine Rolle, und da 4/5 der Masse DM im Halo sein sollen, ist die Abweichung von der Kugelsymmetrie ohnehin nicht sehr groß.

    Die Gravitation nimmt demnach zum Zentrum der Galaxis hin ab, weil die in der entsprechenden Kugelschale eingeschlossene dunkle Materie immer weniger wird. Allerdings nicht linear (wie es bei gleichförmig verteilter DM wäre), sondern weil die DM zum Zentrum hin konzentriert ist, nimmt die Schwerkraft mit geringer werdendem Abstand vom Zentrum erst langsam und dann immer schneller ab.

  64. @Alderamin:

    Klar, dennoch wäre bei dem Modell aber die Zentralmasse BM+DM und dadurch sehr viel größer als durch BM allein. Unser „Nahbereichs“-Gravitationsgesetz ist genau genug. Hinzufügen der DM von 4 x BM würde dieses Gesetz aber sehr in Frage stellen!
    Es reicht auch nicht, einfach die BM auf 1/5 zu verringern und durch DM wieder aufzufüllen, damit die Gesamtmasse wieder stimmt.

    Irgendwie paßt das Modell DM mit Konzentration im Zentrum nicht zum Beobachteten. Wie gesagt: ich denke, dieses Modell ist nur für Simulationen ohne BM zu gebrauchen.

  65. @SCHWAR_A

    Irgendwie paßt das Modell DM mit Konzentration im Zentrum nicht zum Beobachteten.

    Lies‘ Dir mal meinen Link auf Daniel Fischers Bericht über Kroupas Vortrag durch (oben, 8:38), da wird genug aufgeführt, warum DM als Modell besser funktioniert als MOND.

    Ich bin weder Richter noch Zeuge im Prozess DM gegen MOND, nur Beobachter, deswegen möchte ich auch nicht weiter darüber spekulieren. Mir persönlich erscheinen die Argumente für DM jedenfalls plausibler als die für MOND, nach dem was ich so gehört und gelesen habe.

  66. @Alderamin:

    Falls Du die ganze Zeit „MOND“ von M.Milgrom meinst, gebe ich Dir Recht. Die dort formulierte Änderung paßt nicht ganz.

    Die oben von mir formulierte ist da anders und nicht _diese_ „MOND“!
    Aber wenn Du nicht weiter darüber sinnieren möchtest, ist das OK.
    Vielen Dank soweit – bis bald…

  67. @SCHWAR_A

    Ist eigentlich egal von wem die MOND stammt – DM ist halt die derzeit mit Abstand führende Theorie. Wenn sich die Fachwelt von einer bestimmten MOND-Variante überzeugen lässt, übernehme ich das auch gerne (so wie ich die Expansion des Universums und die Entstehung des Mondes durch eine Kollison „geschluckt“ habe – erschien mir beides anfänglich auch nicht sehr glaubwürdig). Aber nicht durch eine Forumsdiskussion – sorry, ich hab‘ zu wenig Ahnung von dem Thema, um hier „richtig“ oder „falsch“ entscheiden zu können.

  68. Heute habe ich Nachbesserungs-Tag: nur die beschleunigte Expansion des Unviersums kam mir anfangs unglaubwürdig vor, als ich das erste Mal von den Messungen hörte. So, wie bei den überlichtschnellen Neutrinos heute.

  69. …aus dt. Wikipedia „Modifizierte_Newtonsche_Dynamik“, Test der Theorie durch Beobachtungen:

    „September 2009 haben Beobachtungen von Famaey und Kollegen gezeigt, dass es für unterschiedliche Galaxien – sowohl kleine Zwergsysteme als auch große Spiralgalaxien – einen starken Zusammenhang zwischen der Verteilung von dunkler und sichtbarer Materie gibt, was wiederum der heutigen Theorie der Dunklen Materie widerspricht. Auch Gianfranco Gentile von der Universität Gent hatte mit seinem Team Berechnungen an 28 Galaxien durchgeführt und stieß auf diesen vermeintlichen Zusammenhang zwischen den Materieformen und äußerte im Fachblatt Nature wieder große Zweifel an der Existenz der Dunklen Materie. Das alles deutet zumindest auf einen noch komplizierteren Zusammenhang von Gravitation und Materie hin.“

    Fazit:

    Diskussion und Kreativität ist hier dringend erforderlich, und warum nicht hier im Forum?

  70. @Alderamin

    …ansonsten lies Dich mal ein wenig in die Keplerschen Gesetze und das Newtonsche Gravitationsgesetz ein

    Für was soll das im diskutierten Zusammenhang gut sein? Selbst Newton – wobei bei den betrachten Massen, einerseits ein BlackHole und andererseits ein WIMP, Newton schon grenzwertig ist – weist klar aus , dass der Gravitationsgradient in Richtung des gemeinsamen Schwerpunkts ausgerichtet ist; zusammen mit ‚cold dark matter‘ ergibt sich doch auch da ein freier Fall genau auf diesen Schwerpunkt zu. Gut wenn jetzt das Schwerezentrum der Milchstraße nicht innerhalb des Zentrums SL liegt…Das ‚hin und her pendeln‘ der DM hat ja auch noch eine andere Konsequenz – um dann noch den hohen Homogenitätsgrad/’Gleichgewicht‘ der Verteilung( und so die Stabilität der Galaxis) zu gewährleisten sollten sich ja sämtliche Bewegungsvektoren der DM – zumindest die, die nicht mit der Rotation von Galaxien zu tun haben – stets aufheben. Von daher kann man sich doch dann auch fragen, ob die DM nicht quasi-stationär mit-rotiert – also auch gar nicht soviel ’ständig von WIMPs durchquert werden‘ ( im Gegensatz zu Neutrinos … mit Ruhemasse ohne Cherenkov-strahlung) stattfindet. Vier-mal die (schwere)Masse der Milchstraße in ständiger Bewegung in einem relativ begrenztem Gebiet gebunden aber doch mit dem Bewegungsvektor der ‚leuchtenden Materie‘ versehen ohne ‚Dichteschwankungen‘? Zumindest sollte eine solche gravitative Bindung mit der ‚Scheibe der leuchtenden Materie‘ gewisse ‚Verwirbelungen‘ in der DM generieren. Kepler 1 … nun wenn da ein WIMP über eine Strecke von mehreren tausend Lichtjahren durch die Galaxis fällt warum soll das dann ein Zweikörperproblem( neben der Frage des nicht durchs Fallen bedingten Bewegungsvektors des WIMP) sein?
    Nicht falsch verstehen, das alles heißt keineswegs, daß ich die Existenz von DM/WIMP per se für zweifelhaft halte aber die Alternative TeVeS (Milgrom) ebenso für nicht von der Hand zu weisen – z.Bsp. der Faktor exp(-n phi) wie auch ‚basteln‘ an der Metrik lassen ja noch Raum zur Modifikation. Könnte ja auch beides(TeVeS und WIMPs) beteiligt sein. Der Charm der DM besteht ja auch darin, dass man nicht nur ‚Wirkung‘ erklärt sondern eben auch gleich noch die Ursache benennt im Gegensatz zu rein geometrischen Erklärungen wie ART und TeVeS.
    Dazu vielleicht noch eine Frage: Ist das Wissen über die Oort’sche Wolke ausreichend um da entsprechende Beobachtungen bzgl. der Gravitationswirkung der Sonne zu bestimmen?

    Fragen eines interessierten Astronomie/Physik Laien.
    Gruß Stefan

  71. @SCHWAR_A

    Wir können hier diskutieren, was wir anderswo gelernt haben, aber keine Lösungen finden oder bewerten, das wäre Dunning-Kruger (jedenfalls was mich betrifft).

    @StefanL

    Selbst Newton – wobei bei den betrachten Massen, einerseits ein BlackHole und andererseits ein WIMP, Newton schon grenzwertig ist – weist klar aus , dass der Gravitationsgradient in Richtung des gemeinsamen Schwerpunkts ausgerichtet ist; zusammen mit ‚cold dark matter‘ ergibt sich doch auch da ein freier Fall genau auf diesen Schwerpunkt zu.

    Auch bei der ISS zeigt der Gravitationsgradient zum Mittelpunkt der Erde und sie bewegt sich trotzdem nicht in diese Richtung. Newton ist sehr wohl relevant, außer man kommt dem Schwarzen Loch im Zentrum der Galaxis bis auf ein paar AE nahe, wo die Schwerkraft so stark wird, dass Einstein eine Rolle spielt. Ein paar AE ist aber schon verdammt nahe, der nächste Fixstern ist über 270 000 AE von der Sonne weg).

    Ich versuch’s nochmal, zu erklären. Ein Schwarzes Loch hat zunächst genau dasselbe Schwerefeld wie ein ausgedehnter, kugelsymmetrischer Körper gleicher Masse, solange man sich außerhalb dieses Körpers befindet. Z.B. könnte man die Sone durch ein kleines Schwarzes Loch von ein paar km Durchmesser ersetzen, dann würde die Erde weiter um sie kreisen. Ein Zusammenstoß mit dem Schwarzen Loch wäre ungleich schwerer als mit der Sonne selbst, weil das Schwarze Loch ein noch viel kleineres Ziel wäre, das man direkt treffen müsste, um reinzufallen; bei der Sonne reicht es, sich dem Zentrum auf weniger als 695000 km zu nähern, ihrem Radius, um in ihrer Atmosphäre zu verglühen. Das Schwarze Loch mit Sonnenmasse könnte man noch in einem Abstand von 10 km umkreisen, ohne reinzufallen.

    Die Gesamtenergie und der Drehimpuls sind zwei Erhaltungsrößen der Orbitalmechanik. Das gilt in Zweikörper- wie n-Körper-Systemen, nur können diese bei n-Körper-Systemen zwischen den Objekten ausgetauscht werden, bleiben aber in Summe erhalten.

    Das bedeutet, wenn ich mich dem Massenschwerpunkt nähere, nimmt meine Bewegungsenergie zu, denn die potenzielle nimmt ab.Ich bewege mich schneller.

    Die andere Erhaltungsgröße ist der Drehimpuls. Wenn ich mich nicht exakt auf das Massenzentrum zubewege, habe ich einen Drehimpuls, rxp: p ist der Impuls, r der Abstand, beides sind Vektoren; wenn r parallel zu p, dann ist das Kreuzprodukt 0, sonst nicht, d.h. ich habe einen Drehimpuls. Den werde ich nicht los, wenn ich auf ein Schwarzes Loch zufalle, und wenn ich bei großem r mit ganz wenig Bewegung, fast stillstehend, aber mit kleiner Geschwindigkeitskomponente nicht parallel zu r, anfange, dann ende ich bei kleinem r in der Nähe des Schwarzen Lochs mit wahnsinnig großem Impuls (Masse*Geschwindigkeit; da sich die Masse nicht verändert, muss es die Geschwindigkeit sein). Das führt mich also dahin, dass ich in der Nähe des Schwarzen Lochs einen rasend schnellen Bogen beschreibe, aber nicht in es hineinfalle, denn das Schwarze Loch muss im Brennpunkt der Bahnkurve sein. Keinesfalls kann es auf der Bahn liegen, weil dann beim Zusammenstoß der Drehimpuls 0 wäre (wegen r=0).

    So, nochmal Zweikörper vs. n-Körper-Problem: Wie gesagt, im n-Körper-System bleiben Drehimpuls und Gesamtenergie insgesamt erhalten. Der DM-Halo der Milchstraße muss, wie die Milchstraße selbst, einen Drehimpuls haben, baryonische und dunkle Materie entstanden ja zusammen und haben sich gemeinsam komprimiert. Die Drehung der Milchstraße zeigt, dass die Gaswolke, aus der sie entstand, einen Drehimpuls hatte.

    Egal, wie die DM miteinander interagiert, diesen Drehimpuls wird sie nicht los. Wenn viel DM in das Schwarze Loch fallen soll, dann muss entsprechend viel nach außen katapultiert werden, sonst stimmt die Drehimpuls-Bilanz nicht. Somit kann das Schwarze Loch im Zentrum der Galaxis nicht den Großteil der DM schlucken. In Wahrheit schluckt es so gut wie gar nichts, weil wegen des oben Gesagten das Schwarze Loch geradlinig getroffen werden muss, um einen Orbit zu vermeiden.

    Besser kann ich’s leider nicht erklären. Hier noch ein Link auf Florians Artikel zu dem Thema =>“Schwarze Löcher sind keine Staubsauger“.

    Dazu vielleicht noch eine Frage: Ist das Wissen über die Oort’sche Wolke ausreichend um da entsprechende Beobachtungen bzgl. der Gravitationswirkung der Sonne zu bestimmen?

    Die Oortsche Wolke ist bisher nicht beobachtbar, wie schließen ihre Existenz nur daraus, dass eine Menge Kometen mit Parabelbahnen durch’s innere Sonnensystem fliegen. Bei Kometen von anderen Sternen würde man erwarten, dass sie eine Anfangsgeschwindigkeit deutlich größer 0 haben, dann würden sie Hyperbelbahnen beschreiben. Bisher ist m.E. noch kein einziger Komet mit einer solchen Bahn entdeckt worden.

    Da wir die Oortsche Wolke nicht sehen können, können wir auch nichts über ihre Bewegung sagen. Und selbst wenn es einen messbaren DM-Effekt gäbe, könnte man immer noch nicht zwischen MOND und DM unterscheiden, die Sonne könnte ja auch einen DM-Halo haben.

    Prinzipiell müsste sich zwischen MOND und DM unterscheiden lassen, wenn man die Bewegung der Kugelhaufen der Milchstraße genau kennen würde, denn DM bildet einen Halo und MOND wäre an die Milchstraßenscheibe gebunden, das Gravitationsfeld wäre nicht dasselbe. Leider sind die Kugelsternhaufen aber so weit weg, dass wir nur ihre Radialgeschwindigkeit messen können, nicht die Eigenbewegung. Somit kennen wir ihre Bahnen nicht genau genug. Sie bewegen sich viel zu langsam.

  72. Alderamin schrieb (09.02.12 · 08:38 Uhr):
    > […] Grundaussage, dass die gemessene Geschwindigkeit im Äther in Längs- und
    Querrichtung nicht dieselbe wäre

    Die Details der oben (09.02.12 · 07:45 Uhr) gezeigten Formeln betreffen aber Beteiligte in einer Region mit „Brechungsindex n“;
    und ihre Aussage ist, dass entsprechende Lichtgeschwindigkeits-Werte „in Längs- und Querrichtung“ insbesondere nur dann einander ungleich sein könnten, falls n ungleich 1 wäre.

    Gibt es denn eine Definition bzw. ein Messprinzip zum „Nachweis von Äther“ kompatibel mit der Versuchsanordnung bzw. dem Befund „n = 1“?

    > Worum es geht […] der Sichtwinkel ist wohldefiniert

    Und zwar entsprechend welcher Definition?

    Sollte man für drei zu betrachtende „Ecken“, A, B und C, und gegebene Distanzverhältnisse zwischen diesen drei Ecken, z.B. die Größe

    ArcSin[ 1/2 Sqrt[
    2 + 2 (AC/AB)^2 + 2 (AC/BC)^2 – (AB/BC)^2 – (BC/AB)^2 – (AC/AB)^2 (AC/BC)^2
    ] ]

    auswerten? …

    > Das betrachtete Dreieck ist gleichschenklig und besteht hier aus der Grundlinie in der kosmischen Hintergrundstrahlung und dem Sichtwinkel beim Beobachter.

    Mit „BC == AC“ ergäbe sich diese Größe wohl als:

    ArcSin[ 1/2 (AC/AB) Sqrt[ 4 – (AC/AB)^2 ] ].

  73. @Alderamin

    zu den Kugelsternhaufen. Ich hab irgendwann in einem SdW etwas zu Zwerggalaxien gelesen, die sich unsere Milchstraße einverleibt hat. Dabei wird dieses kleine Sternsystem zerrissen und (teilweise) zu einer Kette von Sternen. Und genau solche Ketten versucht man zu identifizieren. Das schlaue — und deshalb passt das hierher — daran ist nun, dass diese Sternenkette sich entlang der Bahn dieser Restzwerggalaxie aufreiht, das heißt, dass man dadurch indirekt die Bahn der einzelnen Sterne messen kann und über diese Bahn wiederum auf die Form des Halo der Dunklen Materie schließen kann.

    Müsste suchen wo das war, Schnellsuche mit Google + Nachlesen brachte noch keinen Treffer.

  74. @Frank Wappler

    Zu Michelson-Morley siehe den =>Wiki-Artikel mit den entsprechenden Formeln, da steht nicht, dass es einen Brechungsindex braucht.

    Zum Sichtwinkel: einfach der Winkel, unter dem ein Objekt dem Beobachter am Himmel erscheint. 90° sind es vom Horizont bis zum Zenit, Mond und Sonne haben ca. 1/2 Grad Sichtwinkel. Kennt man aus der Optik, Teleskope vergrößern den visuellen Sehwinkel.

    Zu der Formel: Wenn beim Beobachter A ist und BC die Strecke in der kosmischen Hintergrundstrahlung, dann gilt tan(Sichtwinkel/2)= BC/2 / R, wobei R die Entfernung und Höhe des Dreiecks ist. R ist bekannt, BC ist bekannt, daraus folgt der Sichtwinkel, den man mit der Messung vergleichen kann.

    Für kleine Winkel gilt bekanntlich näherungsweise Phi im Bogenmass * R = BC.

    Bei der kosmologischen Betrachtung kommt noch ein Vergößerungseffekt dazu, weil das Weltall bei der Rotverschiebung (1+z) um den Faktor 1/(1+z) kleiner war, erscheinen Objekte in großer Entfernung wie mit der Lupe vergrößert. Deswegen geht dieser Faktor in die über den Winkeldurchmesser definierte scheinbare Entfernung mit ein (siehe =>hier). Man muss (steht weiter unten) also umgekehrt mit dieser Entfernung R rechnen, um den Effekt der Expansion des Universums bei der Bestimmung des Sichtwinekls auszublenden. (1+z) ist ungefähr 1000 für die kosmische Hintergrundstrahlung.

  75. @Wurgl

    Cool, dann lag ich ja nicht so falsch mit meiner Vermutung. Die Idee mit den auseinandergezogenen Zwerggalaxien ist genial, wenn man dann einen genügend langen Bogen des Orbits zusammenbekommt, um ihn ganz rekonstruieren zu können.

  76. @Myself

    weil das Weltall bei der Rotverschiebung (1+z) um den Faktor 1/(1+z) kleiner war

    Das muss heißen: „…bei der Rotverschiebung z um den Faktor 1/(1+z)…“

  77. Frank Wappler schrieb (10.02.12 · 09:46 Uhr):
    > Mit „BC == AC“ ergäbe sich […]
    > ArcSin[ 1/2 Sqrt[
    > 2 + 2 (AC/AB)^2 + 2 (AC/BC)^2 – (AB/BC)^2 – (BC/AB)^2 – (AC/AB)^2 (AC/BC)^2
    > ] ]

    > wohl als:
    > ArcSin[ 1/2 (AC/AB) Sqrt[ 4 – (AC/AB)^2 ] ].

    Sollte stattdessen sein:
    „Mit „BC == AB“ ergäbe sich …´´.

    (D.h. unter Hervorhebung der „Ecke B“; „B“ wie „Beobachter“).

  78. Hey, hey, hey nicht so schnell mit die Pferde!

    Es kann immer noch sein, dass es DM gar nicht gibt:

    – Dass Phenomene wie der Bullet Cluster eine andere Erklärung haben, zB eine Erklärung als ein Schwarm kleiner schwarzer Löcher, die sich in dem Fall wie DM verhalten haben. Oder

    – Dass es eine Art umgekehrtes MOND Phenomen gibt, wobei die Gravitation in Wirklichkeit viel stärker ist, weil an Materie gebundene zusätzliche Raumdimensionen bisher nicht mit einberechnet wurden, oder es in den „Freiräumen“ des Universums durch eine hyperbolische Raumkrümmung zu einer – in der Wirkung – Anti-Gravitation kommt.

    – Dass wir in einer riesigen Universums-übergreifenden Gravitationswelle leben, die uns zur jetzigen Zeit eine allzu starke Expansion des Universums vorspiegelt. Wenn es die „dunkle Energie“ aber in dem Maße nicht gibt, hat dies sicherlich auch Auswirkungen auf die geschätzten DM Vorkommen.

    – Oder es gibt einen „dunklen Fluß“ des Raumes hin zu Masse Ansammlungen …

    Wenn die Natur es nur „geschickt anstellt“, hat sie große Chancen ihre wahre Natur uns noch recht lange zu verbergen. Im Zweifel länger als unsere forschende Zivilisation halten wird 🙁

  79. Alderamin schrieb (10.02.12 · 10:48 Uhr):
    > Zu Michelson-Morley siehe den =>Wiki-Artikel mit den entsprechenden Formeln

    Dort tritt an prominenter Stelle u.a. der Ausdruck „c + v“ auf, den ich schon bei dir beanstandet habe (s. 09.02.12 · 07:45 Uhr) …

    Was soll dieser Ausdruck und die sich daraus ergebenden Formeln mit der Versuchsanordnung von Michelson-Morley zu tun haben, oder mit deren experimentellen Befunden, oder insbesondere mit der Messgröße „Geschwindigkeit“ ??

    > da steht nicht, dass es einen Brechungsindex braucht.

    Stimmt — das steht da nicht.
    (In Wikipedia steht so Einiges nicht. &)
    Immerhin steht da z.B. auch nicht, dass der Brechungsindex-Wert in der Region mit Michelson-und-Morleys Versuchsanordnung ausdrücklich ganz egal wäre …

    > […] tan(Sichtwinkel/2)= BC/2 / R, wobei R die Entfernung und Höhe des Dreiecks ist.

    Damit betrachten wir i.A. wohl (mindestens) vier unterscheidbare Beteiligte: die drei „Ecken“: A, B und C, sowie einen „Fußpunkt“ („auf einer Seite, gegenüber einer Ecke“).
    Aus deren Distanzverhältnissen lässt sich z.B. bewerten, ob diese vier zueinander „eben“ waren, oder nicht.

    > R ist bekannt, BC ist bekannt, daraus folgt der Sichtwinkel

    Es fragt sich zwar, woher R und BC schon bekannt sein sollten (einzeln, oder wenigstens als Wert des Verhältnisses „BC / R„).
    Aber jedenfalls ist damit die Größe „Sichtwinkel“ auf Distanzverhältnisse zurückgeführt; also offenbar unabhängig von irgendwelchen Vermutungen oder Modell-Annahmen hinsichtlich „Geradheit“ oder „Ebenheit“ oder „Flachheit“ bestimmter Beteiligter untereinander (wie z.B. „Teleskop-Bestandteilen“, oder „Mond-Rändern“ usw.).

    Und wie Distanzverhältnisse zumindest im Prinzip festzustellen sind, ist ja sicherlich nachvollziehbar … vgl. https://en.wikipedia.org/wiki/Metre#cite_note-Res1-0

    > […] den Faktor 1/(1+z)
    > […] die über den Winkeldurchmesser definierte scheinbare Entfernung

    (Ich interessiere mich eigentlich mehr für reelle Messwerte als für Scheinbarkeiten.
    Aber sofern dieser „scheinbare Entfernung“ doch wiederum eine nachvollziehbare Messdefinition zugrundeliegen sollte …)

    Beruht diese Definition (und ggf. sogar einschl. der Definition der Zahl „z“ als Messgröße) nicht auf bestimmten Annahmen, ob und inwiefern die Beteiligten dabei zueinander „(Euklidisch) flach“ waren?

    Dann schiene „scheinbare Entfernung“ doch recht ungeeignet („nicht modell-unabhängig genug“), um im Zusammenhang damit zu bewerten, wer „im Weltall“ zueinander „flach“ war, bzw. inwiefern nicht.

  80. Zitat:
    „Krauss sagt übrigens auch, nur ein flaches Universum kann aus dem Nichts entstehen, weil die bei der Expansion geleistete Arbeit in ihm genau der potenziellen Energie entspricht, den die Materie bei der Expansion erhält.“

    Das verstehe ich als „Laie“ nicht. Kann man das noch genauer erklären bzw. wo anderes „nachlesen“.

  81. @Frank Wappler

    Zu Michelson-Morley:

    Immerhin steht da z.B. auch nicht, dass der Brechungsindex-Wert in der Region mit Michelson-und-Morleys Versuchsanordnung ausdrücklich ganz egal wäre …

    Doch, das steht da, sonst würde der Brechungsindex doch in den Formeln auftauchen.

    Hab‘ mal in meinem Bergmann-Schaefer Teil III (6. Auflage, S. 920 ff) nachgelesen, dass ein Brechnungsindex n=1 schon aus dem Experiment von =>Hoek folgte. Der Versuch von Michelson ergab dann, dass auch für n=1 ein Gangunterschied hätte auftreten müssen vorn Δt= lx*2c/(c²-v²) – ly*2 /√(c²-v²), wobei lx die Länge des Arms des Interferometers in Richtung der Erdbewegung (der klassisch gerechnet einmal mit c+v und einmal mit c-v durchlaufen würde) und ly die Länge des Arms senkrecht dazu (der zweimal mit √(c²-v²) durchlaufen würde). Ich hoffe, dass wir uns darauf einigen können. Wenn nicht – ich beuge mich jedenfalls der Autorität des Lehrbuchs.

    > R ist bekannt, BC ist bekannt, daraus folgt der Sichtwinkel

    Es fragt sich zwar, woher R und BC schon bekannt sein sollten (einzeln, oder wenigstens als Wert des Verhältnisses „BC / R“).
    Aber jedenfalls ist damit die Größe „Sichtwinkel“ auf Distanzverhältnisse zurückgeführt; also offenbar unabhängig von irgendwelchen Vermutungen oder Modell-Annahmen hinsichtlich „Geradheit“ oder „Ebenheit“ oder „Flachheit“ bestimmter Beteiligter untereinander (wie z.B. „Teleskop-Bestandteilen“, oder „Mond-Rändern“ usw.).

    Nein die Formel für das Dreieck beschreibt lediglich, wie in einem euklidischen Universum der Sichtwinkel rechnerisch herauskommen müsste. Das ist die zu verifizierende These. Wenn Weltall flach, dann tan(Sichtwinkel/2)= BC/2 / R. Wenn die Beziehung von der Messung abweicht, dann Weltall nicht flach. Die optische Größe des Sichtwinkels kann ganz ohne Annahmen über irgendeine Geometrie gemacht werden, es ist einfach der Winkel, um den ich ein Teleskop schwenken muss, um vom einen Ende des Objekts zum anderen zum anderen zu schwenken.

    Warum sind R und BC bekannt: Es wird nicht geprüft, ob die Expansionsgeschichte des Universums (Hubble-Parameter in Abhängigkeit von der Zeit nach dem Urknall) korrekt ist, dieser Punkt wird als anderweitig verifizierte Voraussetzung betrachtet. Die kosmische Hintergrundstrahlung entstammt dem Zeitpunkt, als sich Elektronen erstmals mit Kernen zu Atomen formten und damit lichtdurchlässig wurde – das vorherige Plasma war lichtundurchlässig, so wie das Plasma in der Sonne. Dies ist bei ca. 3000K der Fall. Da wir heute ca. 3K als Temperatur der Hintergrundstrahlung messen, ist z = 1000, das Weltall ist seitdem um den Faktor 1000 gewachsen und ebenso die Wellenlänge des Lichts der Hintergrungstrahlung. Aus der Expansiongeschichte folgt, dass dies nach 380000 Jahren der Fall war. Bei einem Weltalter von 380000 Jahren können keine Regionen in Kontakt gekommen sein, die weiter als 380000 Lichtjahre voneinander getrennt sind (es geht hier um sogenannte „Baryonische Oszillationen“, eine Art von Schallwellen im Plasma). Somit ist die größtmögliche Strecke für Korrelationen in der Hintergrundstrahlung 380000 Lichtjahre, und das ist im gedachten Dreieck die bekannte Strecke BC.

    Ebenfalls bekannt ist die Entfernung, da wir unser Weltalter kennen.

    (Ich interessiere mich eigentlich mehr für reelle Messwerte als für Scheinbarkeiten.
    Aber sofern dieser „scheinbare Entfernung“ doch wiederum eine nachvollziehbare Messdefinition zugrundeliegen sollte …)

    Es gibt =>alle möglichen Entfernungsdefinitionen, die alle ihre Berechtigung in bestimmten Zusammenhängen haben. Keine davon ist „die richtige“. Die Winkeldurchmesser-Entfernung ist diejenige, die in einem flachen Universum ohne Expansion aus der Trigonometrie folgen würde. Dieses Entfernungsmaß blendet sowohl die Rotverschiebung als auch nicht-euklidische Geometrien aus.

    Beruht diese Definition (und ggf. sogar einschl. der Definition der Zahl „z“ als Messgröße) nicht auf bestimmten Annahmen, ob und inwiefern die Beteiligten dabei zueinander „(Euklidisch) flach“ waren?

    Ja, das ist korrekt (bis auf den Punkt betreffend z). Ist aber, wie gesagt, Teil der zu verifzierenden These. Wenn das Weltall flach ist, dann gilt für eine auf ein expandierendes Universum verallgemeinerte Winkeldurchmesserentfernung R, dass der oben beschriebene Zusammenhang mit dem Sichtwinkel gelten sollte. Tut er es nicht, ist die These falsifiziert. Und es sei nochmal darauf hingewiesen, dass die Messung dieser sogenannten Baryonischen Oszillationen ausschließlich und nichts anderes als die Ebenheit des Universums belegen soll. Alle zugrundeliegenden Annahmen über die Expansion des Universums und ihre Änderung über die Zeit wurden anderweitig belegt (z.B. über die Entfernungsmessung von Supernovae). Deswegen ist der Einwand unberechtigt, man würde hier Annahmen über z einfließen lassen, die man am Ende in einem Zirkelschluss über die Geometrie belegt.

    Bin aber kein Kosmologe, Niels kennt sich da viel besser aus, vielleicht den mal fragen.

  82. Tom·
    10.02.12 · 23:11 Uhr

    Das verstehe ich als „Laie“ nicht. Kann man das noch genauer erklären bzw. wo anderes „nachlesen“.

    Kann man nachlesen bei =>Lawrence Krauss. Das Argument läuft darüber, dass potenzielle Energie eine negative Energiegröße sei. Es wird erklärt, dass man einen Ball ins Unendliche werfen könne, dann habe er im Unendlichen die potenzielle Energie 0 (am Erdboden hatte eine negative Energie). Wenn die Bewegungsenergie genau so groß sei, dass der Ball im Unendlichen gegen eine Endgeschwindigkeit von 0 strebe, dann habe er in Summe aus Bewegungsenergie und potenzieller Energie stets eine Energie von 0: im Unendlichen sei die potenzielle Energie 0 und die Bewegungsenergie (Ball ruht) ebenfalls 0, also sei die Summe 0. Auf der Erde beim Abwurf sei die potenzielle Energie negativ und die Bewegungsenergie positiv und dem Betrage nach gleich groß wie die potenzielle (der Ball hätte in diesem Fall genau die Fluchtgeschwindigkeit der Erde).

    Das Argument überträgt Krauss nun auf einen Galaxienhaufen, der klein genug sei, dass man die Geschwindigkeiten und Energien „Newtonsch“ berechnen könne, aber groß genug, dass die Expansion des Universums eine Bewegung einer bestimmten Galaxie relativ zum Schwerpunkt des Galaxienhaufens bewirke. Dann gelte auch hier, dass die Summe aus kinetischer und potenzieller Energie genau dann 0 sei, wenn die Galaxie mit gegen 0 strebende Endgeschwindigkeit gerade eben dem Galaxienhaufen entkommen könne.

    Und dann wird Krauss leider etwas vage, weil er die Mathematik vermeiden will: man könne zeigen, dass sich die Newtonsche Formel für die Energiebilanz so umschreiben lasse, dass sie genau die Einsteinsche Gleichung eines expandierenden Universums beschreibe (er meint sicherlich die Einsteinschen Feldgleichungen), und der Term, der die Energiesumme einer Galaxie beschreibe, werde in Einsteins Gleichung zum Term, der die Raumkrümmung des Universums beschreibe. Und nur in einem flachen Universum gelte, dass die Energiesumme genau 0 sei.

    Das ganze werde komplizierter für die ein Universum mit einer Vakuumenergie, die es beschleunige, aber auch da gelte auf hinreichend kleinen Skalen diese Beziehung. Hinzu komme, dass ein Universum, das eine Vakuumenergie habe, Arbeit gegen die Gravitation verrichte, und diese Arbeit sei genau so groß wie die negative Energie, die im räumlich angewachsenen Vakuum als negative Vakuumenergie stecke. Er verweist dann auf Alan Guth.

    Bei Alan Guth (The Inflationary Universe) kann man nachlesen, dass die inlationäre Expansion des Universums auf dem Mechanismus eines negativen Drucks das Vakuums beruhe, der eine abstoßende Gravitation bewirke. Wenn man ein Gas komprimiert und dadurch seinen Druck erhöht, verrichtet man Arbeit, und diese Arbeit steckt als Energie im komprimierten Gas (es wird z.B. heißer). Da Masse äquivalent zu Energie ist, nimmt mithin die Masse und damit die Gravitation zu. Umgekehrt gilt dann, wenn man den Druck verringert, wird die Energie kleiner und nimmt die Gravitation ab. Bei einem expandieren Vakuum führe dies zu einer negativen Energie und abstoßenden Gravitation.

    So geht das Argument. Man muss seine Hirnwindungen ziemlich verknoten, um das alles nachzuvollziehen, und ich kann’s auch nicht besser erklären oder begründen. Ich kann aber nochmal einen Link auf =>Krauss‘ Vortrag hier geben, wo er im Prinzip genau das, was in dem Buch steht, erläutert, und zwar sogar an der Einsteinschen Feldgleichung. Der Vortrag ist zwar lang, aber kurzweilig, da sehr amüsant. Leider in Englisch.

  83. @Alderamin

    Vielen Dank für Deine detailierte Erklärung!
    Ich habe den „Grundgedanken“ verstanden aber mehr ist MIR nicht möglich!
    Das Video von L. Krauss hatte ich schon gesehen, aber daraus nicht verstanden wie er begründet das die Gesamtenergie = 0 ist und das es deshalb möglich ist das ein Universum aus dem nichts entstehen kann.
    Das Video ist wirklich großartig!

  84. @Wurgl & Alderamin:

    „Dabei wird dieses kleine Sternsystem zerrissen und (teilweise) zu einer Kette von Sternen….Müsste suchen wo das war“

    …sucht mal nach „Sternenstrom Kugelstern Haufen“…

  85. Alderamin schrieb (11.02.12 · 12:11 Uhr):
    > lx die Länge des Arms des Interferometers in Richtung der Erdbewegung (der klassisch gerechnet einmal mit c+v und einmal mit c-v durchlaufen würde)

    Du musst im Bergmann-Schäfer sicher nicht weit blättern, um zu sehen, dass stattdessen mit „(c/n + v) / (1 + 1/n v/c)“ bzw. „(c/n – v) / (1 – 1/n v/c)“ bzw.
    „c Sqrt[ 1 – (1 – 1/n^2) (1 – β^2) / (1 + β/n Cos[ φ ])^2 ]“
    zu rechnen ist; sofern es darum geht, Geschwindigkeiten zu berechnen, mit der Signale zwischen zueinander ruhenden Beteiligten ausgetauscht wurden.

    Insbesondere, falls n = 1, nennt man die Geschwindigkeit, mit der Signale ausgetauscht werden (z.B. zwischen zwei „Interferometer-Enden“) „Lichtgeschwindigkeit“, „c“.

    > Es gibt =>alle möglichen Entfernungsdefinitionen, die alle ihre Berechtigung in bestimmten Zusammenhängen haben.

    Dennoch gibt es einen Begriff von (geometrischer) Distanz im eigentlichen, universalen, „properen“ Sinne.
    (Wie z.B. in der Phrase: „Each observer measures their distance to […]“.)

    Seit Einsteins Festsetzungen (bzw. spätestens seit J. L. Synges Aufgreifen der Einsteinschen Definition) versteht sich Distanz in der Physik insbesondere als „Radar distance“; zwischen zueinander ruhenden Beteiligten, und natürlich vorbehaltlich eventueller weiterer Feststellungen betreffs „Brechungsindex“ in der betreffenden Region.

    Und die Nachvollziehbarkeit und Universalität dieser Definition von Distanz als Messgröße spiegelt sich in der (mittlerweile) entsprechend definierten, universellen Maßeinheit: „Meter“.

    > Die optische Größe des Sichtwinkels kann ganz ohne Annahmen über irgendeine Geometrie gemacht werden, es ist einfach der Winkel, um den ich ein Teleskop schwenken muss, um vom einen Ende des Objekts zum anderen zum anderen zu schwenken.

    Die Behauptung, dass sowas einfach und selbstverständlich sei, ist an sich schon eine Annahme, die Geometrie betrifft.
    Das naheliegende Beispiel ist wohl der Begriff „rechter Winkel“, der in der Euklideischen Geometrie offenbar ohne Definition auftaucht.
    Hälst du es etwa für selbstverständlich und ohne irgendeine zugrundeliegende Messmethodik unterscheidbar, ob ein von dir genannter „Schwenk“ „rechtwinklig“ war, oder nicht?

  86. Vorweg:
    Ich bin lediglich interessierter Laie..
    Vor kurzem habe ich mal ein wenig in Unzickers „Vom Urknall zum Durchknall) gelesen.
    Ist seine These, dass die Gravitationskonstante möglicherweise auf so große Objekte, wie Sonnensysteme und Galaxien, nicht anwendbar ist, zu widerlegen?

    Wenn ich es richtig verstehe, dann lässt sich die GK ja nur innerhalb unseres Sonnensystems tatsächlich experimentell bestätigen.

    Die Gravitationskonstante lässt sich schließlich auch nicht auf den Mikrokosmos anwenden. Ist es dann wirklich so absurd, anzunehmen, dass sie auch nicht auf extrem Große Objekte anwendbar ist?

  87. @Markus: „Vor kurzem habe ich mal ein wenig in Unzickers „Vom Urknall zum Durchknall) gelesen“

    Auf das Buch würde ich nichts geben: https://www.scienceblogs.de/astrodicticum-simplex/2010/09/durchgefallen-vom-urknall-zum-durchknall.php

    „Wenn ich es richtig verstehe, dann lässt sich die GK ja nur innerhalb unseres Sonnensystems tatsächlich experimentell bestätigen.“

    Wir können mit den Teleskopen Galaxien sehen, die Milliarden Lichtjahre weit weg sind. Und bis jetzt deutet nichts darauf hin, dass da irgenwelche Konstanten anders sind. Nur weil Unzicker die moderne Physik nicht leiden kann, ist sie deswegen nicht automatisch falsch.

  88. @SCHWAR_A: (Gegoogelt, einen Science-Ticker gefunden und dann vier Hefte durchgeblättert und gefunden)

    SdW 09/2007 „Die Schatten galaktischer Welten“. Und im Kasten auf Seite 56 steht folgender Satz: „Unser Team hat auf diese Weise bereits den Sagittarius-Strom vermessen und dabei herausgefunden, dass die Verteilung der Dunklen Materie in der Galaxis nicht ellipsoid ist, wie von Computermodellen vorhergesagt, sondern kugelförmig.“ Und dann kommt im nächsten Absatz der saukontroverse Satz „Interessanterweise stimmt eine solche Verteilung mit den Vorhersagen alternativer Gravitationstheorien überein, …“.

    Weiter unten wird dann noch geschrieben, dass mit dem Sagittarius-Strom nur eine kleine Region auf Verteilung der DM untersucht wurde und diese Eigenschaft „kugelförmig“ nicht so ganz sicher sei. Jedenfalls gibts einen Satelliten Gaia mit dem man Position und Geschwindigkeit von Sternen in solchen dünnen Strömen so genau vermessen will, dass auch klumpenförmige Ansammlung von DM entdeckt werden könne.

    Der Artikel ist von Rodrigo Ibata und Brad Gibson.

  89. @Frank Wappler

    Wie ich schon sagte steht im Bergmann-Schäfer, dass n = 1 so feststand, als das Michelson-Morley-Experiment stattfand und deswegen taucht auf den Seiten zu Michelson-Morley ff kein n in den Formeln auf, und damit ist für mich dieses Thema beendet. Beschwerden bitte an den Verlag Walter de Gruyter.

    Zu den Entfernungsmaßen: Mit Radar- oder Lasermessung kommst Du in kosmologischen Abständen nicht weit, wenn Du einen Strahl zu einem Objekt am Rande des sichtbaren Universums schicken könntest und auf seine Rückkehr warten, dann käme er niemals wieder zurück, weil das Objekt dem Lichtstrahl enteilt, bevor er es erreicht. Betrachet man nur die Laufzeit des Lichtes von dem Objekt, das uns heute erreicht, so hat dieses eine Strecke c*t zurückgelegt, die zwischen den Entfernungen zum Zeitpunkt des Aussendens und beim Eintreffen des Lichtstrahls auf einer raumartigen Hyperfläche hat (das ist die Mitbewegte Entfernung, sozusagen diejenige, die man hätte, wenn man das Universum einfrieren könnte). Da das Universum aber kontinuierlich expandiert, bekommt man arge Probleme mit der geläufigen „laserbaren“ Entfernung, die wir hier im Kleinen erfolgreich verwenden.

    Zum Thema der Wohldefiniertheit des Sichtwinkels:
    Egal ob der Raum gekrümmt ist oder nicht können wir doch Winkel definieren, die zwei Punkte auf einem gedachten Vollkreis um den Beobachter trennen.
    Fernrohre haben 360° bzw. 24 h-Skalen auf den Achsen und Geodäten messen recht erfolgreich mit ihren Theodoliten Winkelabstände aus. Und in diesem Messraum sind auch Sichtwinkel definiert. 1° ist der 360te Teil eines Vollkreises und hat 60 Minuten zu 60 Sekunden. Und damit kann ich definieren, unter welchem Sehwinkel ein Objekt erscheint. Die Sonne zum Beispiel hat einen Sehwinkel von ca. 0,5°. Damit sage ich nichts über ihre tatsächliche Größeim Verhältnis zu ihrer Entfernung aus – erst bei dieser Rechnung kommt doch die Geometrie ins Spiel.

    Du hast offenbar keine Probleme, bei kosmischen Entfernungen Entfernungsmaße zu verlangen, die sich so wie auf der Erde gar nicht mehr messen lassen, aber auf der anderen Seite bezweifelst du die Wohldefiniertheit von Winkelmessung gerade hier auf Erden? Ich komme da nicht mehr nach, was ist das Problem?

  90. Alderamin schrieb (15.02.12 · 21:16 Uhr):
    > [Es] steht im Bergmann-Schäfer, dass n = 1 so feststand, als das Michelson-Morley-Experiment stattfand

    Bergmann-Schäfer (Band „Optik“) zeigt (auch) ein Bild zum „Aufbau des Michelson-Interferometers„. (In der mir vorliegenden Ausgabe ist das Abb. III, 23, auf S. 325).

    Dort sind in der Versuchsregion insbesondere zwei „Platten“ gekennzeichnet. Wie egal findest du diese Versuchsanordnung, insbesondere hinsichtlich des „Brechungsindex“ in der Versuchsregion (einschl. der als „Platten“ gekennzeichneten Anteile)?

    Und falls das doch nicht ganz egal sein sollte: Wie wurde festgestellt (oder wie wäre zumindest im Prinzip feststellbar), ob (oder zumindest: mit welcher Genauigkeit) die betreffenden Region in den betrachteten Versuchen der Anordnung entsprach?

    > wenn Du einen Strahl zu einem Objekt am Rande des sichtbaren Universums schicken könntest und auf seine Rückkehr warten, dann käme er niemals wieder zurück, weil das Objekt dem Lichtstrahl enteilt, bevor er es erreicht.

    Was ist denn mit den (tatsächlich vorhandenen, oder womöglich zumindest gedanken-experimentell denkbaren) „Objekten am Rande des sichtbaren Universums„, die auf mich (oder dich) „zueilen“;
    und im Zusammenhang mit dem Distanz-Begriff sicher noch wesentlicher:
    Was ist mit denen, die „uns“ gegenüber „starr“ waren, oder eventuell sogar „ruhten“?

    Und wer gehört eigentlich zum „Rand des sichtbaren Universums“ — z.B. die (oben genannten) Atome, von denen „wir, gerade“ beobachten können, dass sie einen Übergang von einem „angeregten“ in ihren „Grundzustand“ anzeigten?

    > Betrachet man nur die Laufzeit des Lichtes von dem Objekt, das uns heute erreicht […]

    Laufzeit des Lichtes“ gilt ja wohl i.A. als Null, „s = 0“; natürlich vorbehaltlich eventueller weiterer Feststellungen betreffs „Brechungsindex“ in der betreffenden Region.
    Vielleicht meinst du ja eine bestimmte andere, von Null verschiedene Dauer, „s < 0"? ... > so hat dieses eine Strecke c*t zurückgelegt, die zwischen den Entfernungen

    … zwischen den — was? …

    > zum Zeitpunkt des Aussendens und beim Eintreffen des Lichtstrahls auf einer raumartigen Hyperfläche hat (das ist die Mitbewegte Entfernung, sozusagen diejenige, die man hätte, wenn man das Universum einfrieren könnte). Da das Universum aber kontinuierlich expandiert […]

    Bleiben wir doch mal bei konkreten, unterscheidbaren Beteiligten („Atome“, „Ecken“, „Enden“ usw.; die beobachtbar sein sollen bzw. die Anzeigen anderer Beteiligter beobachten — d.h. zumindest sofern sie nicht einander zu sehr „enteilen“ — und die jeder für sich ihre eigenen Beobachtungsanzeigen in eine Reihenfolgen ordnen können sollten, so dass von Null verschiedene Werte der Dauer zwischen diesen Anzeigen feststellbar wäre).

    > bekommt man arge Probleme mit der geläufigen „laserbaren“ Entfernung, die wir hier im Kleinen erfolgreich verwenden.

    1. Ich weiß nicht, wie du in diesem Zusammenhang Erfolg bewertest; aber die Voraussetzung für die Bewertung von Erfolg oder Misserfolg einer Messdefinition, und auch für deren eventuelle Geläufigkeit, liegt sicherlich in ihrer Nachvollziehbarkeit.

    2. Sofern z.B. die SI-Einheit „(ein) Meter“ als Distanzwert nachvollziehbar ist, sind offenbar ebenso Distanzwerte wie z.B. „(ein) Yotta-Meter“ oder „(ein) Yocto-Meter“ nachvollziehbar.
    Kleinheit“ oder „Größe“ der ermittelten Werte scheint keine besondere Einschränkung für den Einsatz der SI-Einheit „Meter“ bzw. dem zugrundeliegenden „Einstein-Syngeschen Radar-Prinzip“. Aber:

    3. Problematisch erscheint vielmehr, dass die vorliegende „Meter“-Definition offenbar keinerlei Forderung enthält, dass die gegebenen Beteiligten („Enden“-Paare), deren Distanz zu bewerten ist, dabei zueinander „ruhen“ sollten, bzw. zusammen mit weiteren Beteiligten „flach“ sein sollten, oder wenigstens dabei zueinander „starr“ sollten.
    Geschweige denn irgendwelche Messdefinitionen, wie zumindest im Prinzip zunächst zu entscheiden wäre (ohne den Begriff „Meter“ schon definiert und als Maßeinheit zur Verfügung zu haben), ob oder inwiefern gegebene Beteiligte zueinander „ruhten“ bzw. zueinander „flach“ wären, oder (wenigstens) zueinander „starr“ wären.

    (Um entsprechende Beschwerden muss sich das CIPM wohl nicht sorgen: Physiker interessieren sich sowieso nur für reell-wertige Verhältnisse; sind also gar nicht auf irgendwelche Einheiten angewiesen. Und Ingenieure und Briefmarkensammler, andererseits, kümmert wohl nur das „hier im hinreichend Flachen und Kalten“.)

    > Fernrohre haben 360° bzw. 24 h-Skalen auf den Achsen

    Welche Zahl gehört zu welcher Marke, und warum? …

    > Winkel definieren, die zwei Punkte auf einem gedachten Vollkreis um den Beobachter trennen. […]te Teil eines Vollkreises

    Nennen wir diese beiden „Punkte“ so wie schon oben „A“ und „C“, und „den Beobachter „B“. Und betrachten wir noch zwei Beteiligte, „J“ und „K“ „auf einem (dem selben?) gedachten Vollkreis um“ B.
    Wie soll entscheiden werden, ob „Winkel[ ABC ]“ und „Winkel[ JBK ]“ einander gleich sind bzw. welches reell-wertige Verhältnis sie haben?

    Sollten dafür die Zahlen

    ArcSin[ 1/2 Sqrt[
    2 + 2 (AC/AB)^2 + 2 (AC/BC)^2 – (AB/BC)^2 – (BC/AB)^2 – (AC/AB)^2 (AC/BC)^2
    ] ]

    und

    ArcSin[ 1/2 Sqrt[
    > 2 + 2 (JK/JB)^2 + 2 (JK/BK)^2 – (JB/BK)^2 – (BK/JB)^2 – (JK/JB)^2 (JK/BK)^2
    > ] ]

    ermittelt und miteinander verglichen bzw. ins Verhältnis gesetzt werden?
    Oder was denn sonst?

  91. @Wurgl:

    Hatt’s Dir doch keine Ruhe gelassen, gell? 😉
    Toll und Danke! Hab‘ leider keinen Zugriff darauf – bin kein SdW-Abonnent…:-(

    „Und dann kommt im nächsten Absatz der saukontroverse Satz…“

    Wieso kontrovers? Bei Anwendung von modifizierten Gravitationen ist doch Kugelsymmetrie zu erwarten – also quasi die „Vorhersage“ des jetzt auch Beobachteten…

    Herzliche Grüße.

  92. @SCHWAR_A, Wurgl

    Wieso erwartet man denn Kugelsymmetrie für eine modifizierte Gravitation? Die modifizierte Gravitation ist doch an die Materie gebunden, und wenn es keine dunkle Materie gibt, dann steckt die gesamte Materie in der Scheibe. Hat eine Scheibe denn ein kugelsymmetrisches Gravitationsfeld? Doch eher ein ellipsoides, oder?

    Hingegen erwartet man bei der dunklen Materie doch, dass die Materie keine Scheibe bildet, dann sollte sie doch kugelförmig und nicht ellipsoid verteilt sein? Demnach hätte ich eher erwartet, dass eine kugelförmige Symmetrie für die Dunkle Materie spricht.

    Fragen über Fragen.

  93. „saukontrovers“ weil plötzlich von alternativen Theorien geredet wurde. Ach so. Und ich hab auch kein Abo, aber ein Ikea-Regal 🙂

    … und einen Scanner hab ich auch. Email hat Florian.

    Ich hab auch keine Ahnung, warum diese Computermodelle irgendwelche Ellipsoide haben wollen. Wird auch nicht näher darauf eingegangen. Auf Seite 58 (nur eine Seite) ist dann ein Interview mit Michael Fellhauer/Cambridge University. Der bestätigt die Kugelform durch den Sagittarius-Strom. Der reicht einmal um die Galaxis rum und an der „Naht“stelle sind die Enden ein wenig versetzt. Das soll nur bei sphärisch, also bei Kugel gehen. In dem Interview wird auch kurz angedeutet, dass manche Astronomen eine andere Form als die Kugel haben möchten, aber leider nicht wer und auch nicht warum.

  94. @Alderamin:

    Ja, der Artikel wirft einige Fragen auf. Unten ist ein Doppelbildchen. Da steht was von „Theoretische Vorhersagen“ und dort ist ein ellipsoides Halo gezeichnet mit einem Sternstrom der sich windet (also das Ende schließt nicht mit dem Anfang). Und in der zweiten Hälfte steht „Beobachtungen“ und ein kugelförmiges Halo mit einem Sternstrom der in einer Ebene liegt (das Ende schließt dort mit dem Anfang). Die Diskrepanz ist nun, dass im Interview auf der nächsten Seite die Überschrift „Nur sphärische Halos produzieren ›Gabeln‹“ gewählt wurde und dort wird erklärt, dass eben nur bei kugelförmiger Verteilung dieser Sagittarius-Sternstrom diese Form, nämlich diese offenen Enden haben kann. Erratum find ich keines, allerdings fehlt mit das Dezember-Heft.

  95. @Wurgl

    Auf der Suche nach dem Gravitationsfeld einer Scheibe bin ich auf dieses =>Papier gestoßen, wo es vorgerechnet wird (5.4.3), allerdings nur für einen Punkt auf der Achse der Scheibe. Für den Fall abseits der Achse steht, der Fall würde dann viel komplizierter. Und erst für r->unendlich nähert sich das Feld dem kugelförmigen. Daraus schließe ich mal, dass eine galaktische Scheibe ohne DM kein kugelsymmetrisches Feld haben wird. Jedenfalls nach Newton.

    Wenn eine alternative Gravitation eine andere Entfernungsabhängigkeit hat, wird sie bei Punktmassen trotzdem kugelsymmetrisch sein und bei Scheiben, denke ich, ebenfalls nicht kugelsymmetrisch sein. Müsste man aber mal nachrechnen, um ganz sicher zu sein.

  96. @Wurgl & Alderamin:

    Wenn man sich den Abstand ansieht, den diese blauen DM-Schalen auf den Bildern von der zugehörigen BM haben, erkennt man, wie weit weg von der eigentlichen BM diese zusätzliche DM platziert sein muß. Da kann man den Unterschied zu einem Ellipsoiden wohl nicht wirklich rausmessen, denke ich….

    Obwohl?: Es scheint da _zwei_ Fälle zu geben:

    1. Kugelsternhaufen, die bereits bei etwa r=1.0kpc maximale DM-Dichte aufweisen mit dort ~100-facher Gravitation (NGC8727). (Die sind aber eben IMMER kugelförmig, egal, wo man mißt…)

    2. Galaxien, die erst bei r=1.8Mpc bis zum mehrere-zig-fachen der Gravitation aufweisen können (IC1101). (Das ist etwa der doppelte Radius der Galaxie, also bereits im kugelförmigen Gebiet?…)

    Beide Radius-Bereiche mit ihren Maxima liegen grob Faktor 1800 auseinander und es scheint keine anderen Objekte zu geben, die bei Radien dazwischen eine derart erhöhte Gravitation (oft auch M/L genannt) aufweisen…

    Also hat man es vermutlich mit _zwei_ sich überlagernden DM-Profilen zu tun…

    Ich würde sogar wetten, daß es Sterne innerhalb der Sonnenbahn um das Michstraßenzentrum gibt, bei Abstand ~1kpc, die ebenfalls eine 100-ache Gravitation aufweisen… aber wie könnte man das messen?…

  97. Frank Wappler·
    17.02.12 · 00:26 Uhr

    Bergmann-Schäfer (Band „Optik“) zeigt (auch) ein Bild zum „Aufbau des Michelson-Interferometers“. (In der mir vorliegenden Ausgabe ist das Abb. III, 23, auf S. 325).

    Hab‘ ich auch, auf S. 327.

    Dort sind in der Versuchsregion insbesondere zwei „Platten“ gekennzeichnet. Wie egal findest du diese Versuchsanordnung, insbesondere hinsichtlich des „Brechungsindex“ in der Versuchsregion (einschl. der als „Platten“ gekennzeichneten Anteile)?

    Total egal (solange noch keine Totalreflexion an den Glasplatten auftritt). Da es sich um planparallele Platten handelt und beide abgezweigten Lichtstrahlen je zweimal Platten durchlaufen, sind die Lichtwege gleich, egal welcher Brechungsindex in der Luft herrscht. Wenn das ganz z.B. unter Wasser stattfände, wäre zwar die Brechung an jeder Platte und damit auch die Weglänge durch das Glas eine andere, aber da dies bei beiden Platten, die das Licht jeweils im 45°-Winkel trifft, die gleiche Änderung gegenüber dem Versuch in Luft wäre, wäre die Weglänge in beiden Zweigen immer noch gleich.

    Was ist denn mit den (tatsächlich vorhandenen, oder womöglich zumindest gedanken-experimentell denkbaren) „Objekten am Rande des sichtbaren Universums“, die auf mich (oder dich) „zueilen“;

    Da gibt’s keine, weil die Objekte im Raum ruhen, nur wächst der Raum zwischen uns und ihnen. Die Eigenbewegung lässt sie zwar ein bisschen mehr oder weniger schnell von uns wegeilen, das spielt aber bei großen z keine Rolle mehr.

    und im Zusammenhang mit dem Distanz-Begriff sicher noch wesentlicher:
    Was ist mit denen, die „uns“ gegenüber „starr“ waren, oder eventuell sogar „ruhten“?

    Gibt’s auch keine.

    Und wer gehört eigentlich zum „Rand des sichtbaren Universums“ — z.B. die (oben genannten) Atome, von denen „wir, gerade“ beobachten können, dass sie einen Übergang von einem „angeregten“ in ihren „Grundzustand“ anzeigten?

    Optisch: ja. Theoretisch könnten wir Neutrinos oder Gravitationswellen noch von ein bisschen weiter nachweisen. Der kosmologische Horizont ist die Entfernung von uns, von der aus sich Information, die sich zur Zeit des Urknalls auf den Weg zu uns machte, gerade jetzt erreicht. Es gibt Objekte, die sind noch weiter weg, aber ihr Licht hat uns noch nicht erreicht. Siehe auch =>hier und =>hier.

    „Laufzeit des Lichtes“ gilt ja wohl i.A. als Null, „s = 0“; natürlich vorbehaltlich eventueller weiterer Feststellungen betreffs „Brechungsindex“ in der betreffenden Region.
    Vielleicht meinst du ja eine bestimmte andere, von Null verschiedene Dauer, „s

    Ich meine die Zeit in unserem Intertialsystem, die das Licht von der Aussendung (z.B. kurz nach dem Urknall vor 13,7 Milliarden Jahren) bis zu uns gebraucht hat, und die ist garantiert nicht 0.

    … zwischen den — was? …

    Entfernung in raumartigen Hyperflächen. Wenn der Raum eine Krümmung hat, dann kann man ihn sich als die (raumartige) Oberfläche eines Hyperraums denken. Die Entfernung ist also entlang der Krümmung des Raums gemessen. Und zwar denkt man sich die Zeit dabei eingefroren. Ich habe mich allerdings beim Entfernungsbegriff geirrt, es geht um die Proper Distance, das ist die Entfernung, die man zu einer bestimmten Zeit des Universums zwischen zwei Punkten messen würde, wenn man die Zeit anhalten könnte. Die Mitbewegte Entfernung ist die Proper Distance zur heutigen Zeit und wird als für alle Zeiten unveränderlich definiert. So kann man die Expansion des Universums ausblenden.

    Bleiben wir doch mal bei konkreten, unterscheidbaren Beteiligten („Atome“, „Ecken“, „Enden“ usw.; die beobachtbar sein sollen bzw. die Anzeigen anderer Beteiligter beobachten — d.h. zumindest sofern sie nicht einander zu sehr „enteilen“ — und die jeder für sich ihre eigenen Beobachtungsanzeigen in eine Reihenfolgen ordnen können sollten, so dass von Null verschiedene Werte der Dauer zwischen diesen Anzeigen feststellbar wäre).

    Wird =>hier genau beschrieben.

    1. Ich weiß nicht, wie du in diesem Zusammenhang Erfolg bewertest; aber die Voraussetzung für die Bewertung von Erfolg oder Misserfolg einer Messdefinition, und auch für deren eventuelle Geläufigkeit, liegt sicherlich in ihrer Nachvollziehbarkeit.

    Z.B. dass mein Laser- oder Radarstrahl in unendlicher Zeit nicht mehr zurückkommt, würde ich als erfolglose Messung bezeichnen.

    2. Sofern z.B. die SI-Einheit „(ein) Meter“ als Distanzwert nachvollziehbar ist, sind offenbar ebenso Distanzwerte wie z.B. „(ein) Yotta-Meter“ oder „(ein) Yocto-Meter“ nachvollziehbar.

    Jo, aber in welchem Entfernungsmaß? Proper distance, Lichtlaufzeit-Entfernung (das ist, anders als im deutschen Wiki-Artikel angegeben, nicht dasselbe), Mitbewegte Entfernung, Winkeldurchmesserentfernung…

    3. Problematisch erscheint vielmehr, dass die vorliegende „Meter“-Definition offenbar keinerlei Forderung enthält, dass die gegebenen Beteiligten („Enden“-Paare), deren Distanz zu bewerten ist, dabei zueinander „ruhen“ sollten, bzw. zusammen mit weiteren Beteiligten „flach“ sein sollten, oder wenigstens dabei zueinander „starr“ sollten.

    Jo, das ist das Problem in einem expandierenden Universum. Zumindestens bei Proper Distance und Comoving Distance „tut man so“, als ob die Endpunkte starr seien.

    „Fernrohre haben 360° bzw. 24 h-Skalen auf den Achsen“
    Welche Zahl gehört zu welcher Marke, und warum?…

    24h hat die Stundenachse (wer hätte das gedacht), die parallel zur Erdachse ist, weil sich die Erde in 24h (siderischen) Stunden einmal dreht, also folgt aus dem Winkelabstand gleich die Driftzeit, wenn man das Fernrohr anhält und das Objekt durch die Erddrehung durchs Blickfeld driften lässt. +90°…-90° misst man entlang der Deklinationsachse, die den Breitengraden am Himmel entspricht. Winkelabstände zwischen zwei Objekten misst man aber generell in Grad-Minuten-Sekunden auf der gedachten Himmelskugel.

    Nennen wir diese beiden „Punkte“ so wie schon oben „A“ und „C“, und „den Beobachter „B“. Und betrachten wir noch zwei Beteiligte, „J“ und „K“ „auf einem (dem selben?) gedachten Vollkreis um“ B.
    Wie soll entscheiden werden, ob „Winkel[ ABC ]“ und „Winkel[ JBK ]“ einander gleich sind bzw. welches reell-wertige Verhältnis sie haben?

    Mit der sphärischen Trigonometrie kann ich auf der gedachten Himmelskugel die Winkelabstände zweier beliebiger Punkte berechnen. Man übertrage einfach die Himmelskoordinaten der Objekte auf eine Kugel mit dem Radius 1 und berechne die Länge des Großkreissegements, das man durch die beiden Punkte (einmal AC, einmal JK) legen kann. Et voila.

    Sollten dafür die Zahlen
    […]
    ermittelt und miteinander verglichen bzw. ins Verhältnis gesetzt werden?

    Mag sein, bin aber jetzt zu faul, das herzuleiten. Sphärische Trigonometrie und Großkreis, halt.

  98. Alderamin schrieb (17.02.12 · 20:54 Uhr):
    > [das Michelson-Morley-Experiment … in der Versuchsregion insbesondere zwei „Platten“] Total egal (solange noch keine Totalreflexion an den Glasplatten auftritt).

    Schon allein die Sicherstellung dieser Forderung („keine Totalreflexion„) beinhaltet die Bewertung der Brechungsindex(-Verhältniss)e in der Versuchsregion. Deine oben (15.02.12 · 21:16 Uhr) genannte Bedingung „n = 1“ scheint weder gefordert noch zwangsläufig gewährleistet.

    > Da es sich um planparallele Platten handelt

    Offenbar ist es dir auch alles andere als egal, ob der Brechungsindex „in der einen Platten-Region“ homogen wäre, und der Brechungsindex „in der anderen Platten-Region“ homogen wäre, und der Brechungsindex „außerhalb der Platten“ homogen wäre.

    > und beide abgezweigten Lichtstrahlen je zweimal Platten durchlaufen, sind die Lichtwege gleich, egal welcher Brechungsindex in der Luft herrscht.

    Womöglich ist es dir ja nicht mal egal, ob die Brechungsindices „in den beiden Platten-Regionen“ einer gleich waren (bzw. während des Versuches gleich geblieben wären), und mit welchen Geschwindigkeiten sich Systeme innerhalb diese „Platten-Regionen“ gegenüber einander bewegten, die jeweils Lichtgeschwindigkeit als „in alle Richtungen gleich“ bewerten würden.

    Besonders interessant ist aber die Bemerkung betreffend „Gleichheit der Lichtwege„.
    Wurde das im Rahmen der Versuchsanordnung des Michelson-Morley-Experiments gefordert?
    Und falls so, wie wurde nachgewiesen (bzw. wie wäre nachzuweisen), ob diese Forderung erfüllt wurde — etwa durch Forderung und Feststellung unveränderter Intereferenzmuster während des Versuches?

    > [tatsächlich vorhandene, oder womöglich zumindest gedanken-experimentell denkbare „Objekten am Rande des sichtbaren Universums“, die auf mich (oder dich) „zueilen“] Da gibt’s keine

    Hmm … Um mal bei dem (bzw. denjenigen) zu bleiben, das (bzw. die) es gibt:
    Für jedes Paar von zueinander zeitartigen Ereignissen sind doch wohl Beteiligte zumindest denkbar (wenn nicht sogar auffindbar), die an diesen beiden Ereignissen beteiligt waren? (Oder etwa nicht?)

    > [Bleiben wir doch mal bei konkreten, unterscheidbaren Beteiligten …] Wird =>hier genau beschrieben.

    Soweit waren wir oben (15.02.12 · 00:05 Uhr) schon mal — Was bedeutet „distance=>hier: „Each observer measures their distance to […]“ ??

    > Man übertrage einfach die Himmelskoordinaten der Objekte auf […]

    Koordinaten?? Geometrische Beziehungen (Distanzverhältnisse, Winkel, Geschwindigkeiten, usw.) zwischen unterscheidbaren Beteiligten sind ja wohl unabhängig davon, ob und welche Koordinatenzahlen darüber gestreuselt wurden.

  99. Ich habe doch mal eine Frage die dunkle Energie betreffend: Sie ist ja die Folge der Beobachtung, dass sich das Universum zunehmend schnell ausdehnt. Wenn man sich vorstellt, dass der Urknall von einem Punkt ausgegangen ist, müsste man aufgrund der Masse erwarten, dass sich die Ausdehnung verlangsamt, also muss da eine Energie für die Beschleunigung sorgen. So weit, wie ich es verstanden zu haben glaube.

    Jetzt meine Frage: Kann es nicht sein, dass sich das Universum immer noch mitten in der Phase der „Explosion“ befindet, in der die Teile zunehmend durch eben dieses Ereignis selbst beschleunigt werden? Das würde heißen, dass der Zeitpunkt der Verlangsamung trotz der vielen Milliarden Jahre einfach noch nicht erreicht wäre, weil diese Explosion kosmologisch vielleicht noch so jung ist und sich in einer Frühphase befindet.

    *mich warm anzieh*

  100. Hallo zusammen,
    bin durch Zufall auf diesen Blog gestoßen.
    Ist ja irre was da an Wissen und Formeln geboten wird.
    Ich bin ja nur Maschinenbauer und hab in meinem Studium
    nur die normale Physik gehabt.
    Allerdings weiss ich eines:
    Wenn ich mit demm Hammer auf den Amboss haue entsteht
    Wärme, d.h. es wird Energie frei.
    Kinetische Energie verwandelt sich, aber es geht nichts verloren.
    Wir nutzen die Energie der Sonne um aus „Licht/Strahlung“
    oder sonst was Energie zu gewinnen und es funktioniert.
    Nachdem diese Energiegewinnung nicht durch Photosynthese
    funktioniert, sollte meiner Meinung nach eine gewisse
    „Masse“ vorhanden sein um nach den einfachen Prinzipien
    eine Energieabgabe zu gewährleisten.
    Hört sich vielleicht recht einfältig an, aber warum sollte etwas das
    keine „Masse“ besitzt an irgend etwas anderes Engerie abgeben ?,
    und noch dazu freiwillig ?, Dann wäre Tesla wieder aktuell.
    Sollte Licht dann doch in irgend einer Form „Masse“ sein, wäre es
    natürlich interessant zu wissen, ob es was gibt das „leichter“ ist als
    Licht und damit auch schneller sein kann.

    Wenn Ihr antworten wollt, macht es bitte nicht zu kompliziert,
    ich habe keine Lust mir die alten Formelsammlungen raus zu suchen
    oder in meinem Alter noch ein Studium zu absolvieren.

  101. Nanü?
    Noch keine Antworten eingetrudelt?
    Komisch, na gut, dann will ich mich mal versuchen, auch wenn ich in Physik nicht so irre Sattelfest bin…

    @Fränki

    und hab in meinem Studium nur die normale Physik gehabt.

    Gibt es denn abnormale Physik im Studium? Meines Wissens gibt es da nur eine Physik, sonst nix.

    Allerdings weiss ich eines:
    Wenn ich mit demm Hammer auf den Amboss haue entsteht
    Wärme, d.h. es wird Energie frei.

    Nee, ich befürchte, das weißt Du nicht so ganz richtig. Da wird ganz bestimmt keine „Energie frei“. Aber vielleicht drückst Du Dich da auch nur unpassend aus, denn danach schreibst Du von Umwandlung von kinetischer Energie. Das ist schon richtiger, aber verloren geht hier trotzdem eine ganze Menge.

    Wir nutzen die Energie der Sonne um aus „Licht/Strahlung“ oder sonst was Energie zu gewinnen und es funktioniert.
    Nachdem diese Energiegewinnung nicht durch Photosynthese
    funktioniert, sollte meiner Meinung nach eine gewisse
    „Masse“ vorhanden sein um nach den einfachen Prinzipien
    eine Energieabgabe zu gewährleisten.

    Hm…also da bin ich mir nicht so ganz sicher, was Du jetzt damit meinst. Und wieso schreibst Du „Licht/Strahlung“?
    Das dazwischen lasse ich mal alles aus, aber das hier:

    …wäre es natürlich interessant zu wissen, ob es was gibt das „leichter“ ist als Licht und damit auch schneller sein kann.

    Vielleicht liest Du Dir erst mal diese Seite durch:
    Wie schwer ist Licht?

    Dort wird das schön erklärt. Danach sollten einige Deiner Fragen schon mal eher geklärt sein.
    Falls das Dein Interesse geweckt hat, dann könntest Du im Anschluss auch noch diesen Artikel lesen:
    Warum lautet Einsteins Formel E = mc^2 und nicht E = mc?

  102. @Dietmar

    Lese Deine Frage erst jetzt.

    Kann es nicht sein, dass sich das Universum immer noch mitten in der Phase der „Explosion“ befindet, in der die Teile zunehmend durch eben dieses Ereignis selbst beschleunigt werden? Das würde heißen, dass der Zeitpunkt der Verlangsamung trotz der vielen Milliarden Jahre einfach noch nicht erreicht wäre, weil diese Explosion kosmologisch vielleicht noch so jung ist und sich in einer Frühphase befindet.

    Nein, die Verlangsamung aufgrund der wechselseitigen Gravitation fand in der Anfangsphase des Universums statt, als die Materie viel dichter war, und nahm mit abnehmender Dichte immer mehr ab, bis die Beschleunigung durch die stets konstante Vakuumenergie vor ca. 5 Milliarden Jahren die Oberhand gewann. Die Expansionsgeschichte des Universums lässt sich anhand von Entfernungsmessungen mit Supernovae vom Typ Ia direkt beobachten. Siehe z.B. auch diesen Artikel.

  103. Beide Phänomene wurden entdeckt !!!
    … wir nicht wissen, worum es sich bei der dunklen Materie handelt
    Es gibt eine Vielzahl sehr handfester Beobachtungsdaten, die die Existenz der dunklen Materie nahelegen

    ALSO , ES WURDE ETWAS ENTDECKT , WIR WISSEN ABER NICHT WORUM ES SICH DABEI HANDELT , UND DESSEN EXISTENZ WURDE DURCH HANDFESTE BEOBACHTUNGSDATEN NAHEGELEGT .

    Klingt nicht gerade überzeugend , oder ?
    Von wissenschaftlich bewiesen kann überhaupt nicht die Rede sein . Reine Spekulation !

    1. @uwe: Was genau ist denn deiner Meinung nach „reine Spekulation“? Die bald seit 100 Jahren stattfindenden Beobachtungen die zeigen, dass sich Sterne, Galaxien und Galaxienhaufen anders bewegen als sie es rein unter dem Einfluss der sichtbaren Materie tun?

Schreibe einen Kommentar zu Alderamin Antworten abbrechen

Deine E-Mail-Adresse wird nicht veröffentlicht. Erforderliche Felder sind mit * markiert

Diese Website verwendet Akismet, um Spam zu reduzieren. Erfahre mehr darüber, wie deine Kommentardaten verarbeitet werden.